Космос и астрономия


Ответить в тред Ответить в тред

Check this out!
<<
Назад | Вниз | Каталог | Обновить тред | Автообновление
527 47 164

Тред тупых вопросов №104 HUBBLE EDITION Аноним 19/03/19 Втр 02:29:57 4713831
KU2.0.6Hubbleim[...].jpg (40Кб, 520x700)
520x700
opo0524a.jpg (1702Кб, 2868x2294)
2868x2294
9724238.gif (3Кб, 444x345)
444x345
videoplayback.webm (1388Кб, 1280x720, 00:00:30)
1280x720
Тред вопросов о жизни, Вселенной и всём таком.

Спрашиваем то, за что в других местах выдают путёвку в биореактор. Здесь анонимные учёные мирового уровня критически рассмотрят любые гениальные идеи и нарисованные в Paint схемы.

Предыдущий тут: >>464488 (OP)
https://2ch.hk/spc/res/464488.html

Q: Можно быстрее?
A: Можно упасть в пузырь альбукерке, наса уже почти надула его.

Q: Я начитался охуительных историй про уфологию, че делать, нам жопа?
A: Да, тебе жопа, можешь сгонять в зогач или куда оттуда пошлют.

Q: Что будет с человеком в вакууме без скафандра / если он упадет на черную дыру / попробует ступить на поверхность газового гиганта/солнца?
A: Он умрёт.

Q: Почему бы не привязать ракету к воздушному шару или стартовать с горы?
A: Космос - это не как высоко, а как быстро, большая часть энергии ракеты уходит на разгон вбок.
Подробнее тут https://what-if.xkcd.com/58/ (английский) https://chtoes.li/orbital-speed/ (перевод)
Аноним 19/03/19 Втр 02:40:30 4713852
(все спиздил у Леметра)
Аноним 19/03/19 Втр 06:47:17 4713953
>>471383 (OP)
Что на 3 пике?
И да:
Ебанный стыд...
Во-первых, Алькубьерре.
Во-вторых, не упасть, а создавать вокруг корабля изнутри (иначе кина не будет).
В-третьих, НАСА искривляет пространство на десятимиллионную часть, контролируя это сверхточными интерферометрами, до самого варп-привода здесь - как до Антарктиды раком.
Аноним 19/03/19 Втр 07:02:13 4713964
>>471395
Хаббловская диаграмма (зависимость скорости удаления (в км/с), измеренной по красному смещению, от расстояния (в Мпк)) для сверхновых типа Ia. Тангенс угла наклона прямой, аппроксимирующей эту зависимость, дает современное значение постоянной Хаббла 64 км/с/Мпк.
Аноним 19/03/19 Втр 09:38:15 4714315
Зачем батареи для МКС нужно было устанавливать снаружи, да причем так, что для их замены нужно выходить в открытый космос каждый раз? Видео релейтед
https://www.youtube.com/watch?v=i-V_PhACTJw
Аноним 19/03/19 Втр 10:21:10 4714356
>>471383 (OP)
Ебанный стыд...
Во-первых, Алькубьерре.
Во-вторых, не упасть, а создавать вокруг корабля изнутри (иначе кина не будет).
В-третьих, НАСА искривляет пространство на десятимиллионную часть, контролируя это сверхточными интерферометрами, до самого варп-привода здесь - как до Антарктиды раком.
Аноним 19/03/19 Втр 10:37:33 4714387
Аноним 19/03/19 Втр 11:12:17 4714438
>>471438
Легимитизация перекота жи
Аноним 19/03/19 Втр 11:36:39 4714459
>>471431
А зачем им внутри быть, если им и в вакууме норм?

Герметичный объем не бесконечный все-таки, если эти панели дергают даже не раз в год, гораздо разумнее повесить их снаружи.
Аноним 19/03/19 Втр 11:40:09 47144610
>>471445
и каждый раз вылезать наружу? это гемморой лишний и там они уязвимы для микрометеоритов.
Аноним 19/03/19 Втр 12:04:43 47145011
1.jpg (84Кб, 852x480)
852x480
Аноним 19/03/19 Втр 13:01:34 47146012
>>471450
Возможно да , а может нет.
Аноним 19/03/19 Втр 14:09:46 47146813
>>471446
Космонавты и так вылезают наружу каждый месяц-два, от добавления плановой замены батарей раз в несколько лет сильно хуже не станет.

Внутри МКС и все пространство забито разной хуйней и свободного места не шибко много. Если что-то нормально себя чувствует за бортом и не требует частого обслуживания, там ему самое и место.
Аноним 19/03/19 Втр 17:53:40 47152614
Аноним 19/03/19 Втр 20:08:58 47155015
Предположим, что у нас есть два корабля, висящие рядом. На одном установлен плазменный электроутюг на пердячьем паре мощностью 10 кВт. Мы включаем его в розетку, и корабль начинает ускоряться, скажем, на 0,01 g (числа не важны). Относительно второго корабля его скорость растет линейно по времени, а значит кинетическая энергия - пропорционально квадрату времени. Но ведь утюг потребляет постоянную мощность из розетки! Как получается, что тратим мы одну и ту же энергию, а прирост все больше и больше?
Аноним 19/03/19 Втр 20:32:48 47155116
>>471550
Потому что рабочее тело, которые ты расходуешь, вместе с кораблём летит всё быстрее и быстрее и его импульс растёт.
Аноним 19/03/19 Втр 20:55:24 47155917
>>471550
Недостающая энергия извлекается из кинетической энергии самого топлива. До того, как попасть в двигатель, топливо же движется вместе с кораблем и имеет ненулевую кинетическую энергию, и чем быстрее летишь, тем больше энергии из него достается.

Рассмотри например, такой случай, когда корабль уже летит со скоростью истечения топлива (назовем ее v). Тогда топливо внутри корабля изначально имеет кинетическую энергию mv²/2, а выброшенные из двигателя продукты горения будут иметь нулевую скорость относительно наблюдателя и нулевую кинетическую энергию. Все эти самые mv²/2 были полностью переданы кораблю.

Если корабль движется дохуя быстро, например, в десять раз быстрее, чем в прошлом примере, то ситуация еще явнее. Топливо внутри корабля уже имеет скорость не mv²/2, а 100×mv²/2, а продукты сгорания — 81×mv²/2, т.е. топливо передало кораблю в 19 раз больше энергии, чем в предыдущем случае.
Аноним 19/03/19 Втр 20:57:05 47156218
>>471559
>Топливо внутри корабля уже имеет кинетическую энергию не mv²/2, а 100×mv²/2
фикс
Аноним 19/03/19 Втр 21:06:47 47156419
>>471551
>>471559
А нельзя это просто объяснить тем, что корабль расходует топливо, то есть линейно уменьшает массу, а значит кинетическая энергия растёт всё равно линейно?
Аноним 19/03/19 Втр 21:17:18 47157020
>>471564
Нет, вся суть в том, что кинетическая энергия не растет линейно.
Аноним 19/03/19 Втр 21:51:33 47158021
>>471559
То есть суммарная энергия всей системы корабль + топливо остается неизменной?
Аноним 19/03/19 Втр 22:00:55 47158122
>>471580
Законы сохранения пока еще работают, как видишь.
Аноним 19/03/19 Втр 22:04:17 47158223
Аноним 19/03/19 Втр 22:04:31 47158424
>>471581
Хорошо, а как быть в такой ситуации. Однояйцевые близнецы Вася и Петя висят рядом с кораблем. Общий импульс системы Вася + Петя нулевой и у каждого нулевая кинетическая энергия относительно корабля. Потом они отталкиваются и разлетаются в стороны со скоростью 1 м/с. Импульс по-прежнему нулевой, это понятно. Но у каждого появилась кинетическая энергия относительно корабля. Откуда?
Аноним 19/03/19 Втр 22:06:58 47158525
>>471580
>То есть суммарная энергия всей системы корабль + топливо остается неизменной?
Она возрастает. Но она в каждый момент времени возрастает ровно настолько, сколько было взято из розетки.
Аноним 19/03/19 Втр 22:09:31 47158826
>>471584
>Но у каждого появилась кинетическая энергия относительно корабля. Откуда?
Из превращения химической энергии съеденной Петей/Васей нямки в работу мышц во время отталкивания.
Аноним 19/03/19 Втр 22:19:54 47158927
>>471588
Все, понял. Ща расписал пример с кораблем через Васю и Петю (с условием, что изначально они летят со скоростью n м/с), и получилось, что прирост суммарной кинетической энергии всегда одинаков. Ну а из нямки или розетки - не важно.
Аноним # OP 20/03/19 Срд 04:34:40 47163128
Довольно тупой вопрос по поводу перекатов: а шебмки из шапок тредов вообще кто-нибудь смотрит, или от их отсутствия ничего страшного не произойдет?

Просто в данный момент сложилась более-менее стабильная cхема шапки: портрет ученого, две картинки, имеющие отношение к его научной работе, релевантная анимация. Портреты и картинки найти легко, а вот уместные анимации подобрать гораздо сложнее: в итоге поиск и времени занимает, и все равно мне кажется, что какая-то хуйня получается.

Может, и без шебмок норм, хватит и просто портрета и трех картинок, или с анимациями лучше?
20/03/19 Срд 09:11:40 47167529
>>471631
Совершенно пофиг, мне кажется. Нашёл -- отлично, не нашёл -- да никто и внимания не обратит. Говорю не в том плане, чтобы принизить твою работу, разумеется. Я раз 10 перекатывал SpaceX-тред (в 16-м году), и замучался искать нормальные неповторяющиеся картинки, а ты про видео говоришь.
Аноним 20/03/19 Срд 10:20:29 47168330
>>471631
Я всегда смотрю и пикчи и шебм, и от них немного бугурчу. Вот сейчас тоже у меня немного бугурт от 3-й пикчи и вебм. Ящитаю без вебм будет лучше, так я с меньшей вероятностью зайду в тред с целью разъебать очередной тупой ответ наивного релятивиста.

мимокефир
Аноним 20/03/19 Срд 10:22:32 47168431
>>471683
>бугурчу
>мимокефир
Все правильно делаешь.
Аноним 20/03/19 Срд 10:49:00 47168732
>>471383 (OP)
Пиздоболия. Как раз большая часть топлива уходит на старте чтобы пройти через плотные слои атмосферы. Если стартовать там где повыше, то атмосфера не будет мешать и для разгона потребуется значительно меньше топлива.
Аноним 20/03/19 Срд 12:55:52 47170833
>>471687
Она и не мешает, аэродинамические потери за весь полет составляют в районе 100 м/с. Из 9500 общей дельты, требуемой для выхода на орбиту.
Аноним 20/03/19 Срд 12:56:36 47170934
15498818558340.jpg (80Кб, 828x746)
828x746
А что если проводить физические опыты близко к горизонту событий у ЧД, но при этом наблюдать за ними на удаленном расстоянии? Может что нибудь засечем новое, пока они там слоупочить будут?
Аноним 20/03/19 Срд 13:06:21 47171235
>>471709
Ага, завтра попробуем. ЧД доступную найди. Да еще сверхгигантскую, чтобы возле ее горизонта событий еще не наступала спагеттификация. И еще придумай как наблюдать эксперимент в сверхдлинных радиоволнах с охуенной точностью - ведь если у тебя такое релятивистское замедление, то и свет должен в красную область пиздец как далеко уйти.
Аноним 20/03/19 Срд 13:44:44 47171936
15498818558340.jpg (80Кб, 828x746)
828x746
>>471712
В плане бреда и сайфаяА если у горизонта событий поставить клетку с колибри, которая летает в ней. А мы будет снимать видео на "безопасном" расстоянии, что мы, в теории, увидим? видео с замедленными движениями? Обрезки видео? Ничего не увидим? Обычное видео?
Аноним 20/03/19 Срд 13:50:50 47172437
>>471708
Это потому что сначала ракета поднимается под большим углом, а потом уже ложится на баллистическую траекторию. Именно потому что атмосфера внизу мешает сразу разгоняться до орбитальной скорости
Аноним 20/03/19 Срд 14:37:01 47172838
Если бы Марс и Венера вращались бы вокруг Солнца по той же орбите что и Земля их было бы легче терраформировать?
Допустим Венера вращается с той же скоростью что и Земля, но с отставанием на полгода по орбите, а Марс поставить вместо Луны, чтобы его защищала магнитосфера Земли, а Луну отдать Венере. Получим ли мы сразу три обитаемые планеты?
Аноним 20/03/19 Срд 14:51:26 47173139
>>471719
>А если у горизонта событий поставить клетку с колибри, которая летает в ней.
Ты не сможешь ее там поставить. Она должна будет лететь по орбите вокруг ЧД с почти световой скоростью, иначе провалится. И дыра должна быть сверхмассивной, иначе разорвет все на атомы.
>А мы будет снимать видео на "безопасном" расстоянии, что мы, в теории, увидим?
Очень замедленные движения. И обычный видимый свет очень сместится в инфракрасный спектр или даже радиоволны, в зависимости от близости к ЧД.
Аноним 20/03/19 Срд 15:01:43 47173240
>>471731
>Очень замедленные движения.
Почему это? Движения будут обычными, просто будет большая задержка, как между марсоходом и его операторами на Земле.
Аноним 20/03/19 Срд 15:06:55 47173341
>>471732
Потому что в клетке замедленное время, за счёт скорости и гравитации. А вот на задержке сигнала это как раз особо не скажется.
Аноним 20/03/19 Срд 15:09:56 47173542
>>471728
С Марсом может и прокатит, а вот Венере от этого лучше не станет, а если ей ещё и Луну дать, то может ещё и хуже будет, если она создаст Венере магнитное поле и усилит вулканизм.
Аноним 20/03/19 Срд 16:46:13 47175043
>>471731
>>471732
>>471733
Так, что-то я уже сам запутался. Допустим, колибри внутри клетки чирикает каждый раз, как завершается оборот вокруг ЧД. Как тогда это должно выглядеть снаружи?
Аноним 20/03/19 Срд 16:51:37 47175244
>>471750
А, кажется отпутался. Из-за релятивистского сокращения длины, для колибри путь вокруг ЧД будет короче, чем для внешнего наблюдателя. То есть, по своим часам она будет крутиться быстрее и чирикать чаще. А для внешнего наблюдателя и то, и другое будет происходить медленнее (но все равно довольно быстро, скорость-то почти световая).
Аноним 20/03/19 Срд 19:31:47 47177845
Пейсаны, скиньте статью на википедию об образовании элементов тяжелее железа. Я просто никак не догадаюсь по какому тэгу гуглить.
Аноним 20/03/19 Срд 19:44:14 47178346
Аноним 21/03/19 Чтв 10:19:04 47195547
147144647612451[...].jpg (158Кб, 671x360)
671x360
Помню в детстве читал энциклопедию про космические аппараты. Очень много вроде было там картинок посадочных модулей, ракет.
Там ещё описывалась история про китайца который впервые открыл реактивное движение за счёт привязанных к стулу ракет.
Никто не помнит такой?
Аноним 21/03/19 Чтв 10:45:17 47195948
>>471955
>Там ещё описывалась история про китайца который впервые открыл реактивное движение за счёт привязанных к стулу ракет.
лол че за бред то есть реактивное движение открыли именно когда привязали к стулу, а не когда придумали ракеты?
Аноним 21/03/19 Чтв 12:05:58 47197849
>>471955
Этим китайцем был Вань Ху.
Аноним 21/03/19 Чтв 12:13:33 47198050
Поневнимательности запостил в старый тред. Перенесу сюда.

Ещё вопрос по орбите. Если орбита спутника наклонена относительно оси Земли, при этом направление движения спутника совпадает с направлением вращения Земли, то что случится, если начать корректировать курс спутника, увеличивая наклон и пройдя таким образом в какой-то момент ось вращения Земли? Спутник будет двигаться в противоположную от направления вращения земли сторону?
Аноним 21/03/19 Чтв 12:17:24 47198151
>>471728
>а Марс поставить вместо Луны, чтобы его защищала магнитосфера Земли
У меня сразу вопрос о приливных силах возник. Это же будет катастрофично?
Аноним 21/03/19 Чтв 12:38:46 47198652
>>471980
Бля, да установи ты уже огурцов.
Аноним 21/03/19 Чтв 14:02:00 47201353
>>471728
>Допустим Венера вращается с той же скоростью что и Земля, но с отставанием на полгода по орбите
Температура на Венера от этого почти не изменится, чтоб с текущей атмосферой температура была на уровне земной ее надо чуть ли не к Юпитеру задвинуть.
>Луну отдать Венере
Зачем, если у нее есть своя Луна Меркурий
Аноним 21/03/19 Чтв 14:33:29 47202154
Не забывайте еще, что Венеры и Земли на одной орбите не могло быть, такая система нестабильна. Рано или поздно резонанс бы разрушился, и либо одну из планет выкинуло бы нахуй на другую орбиту, либо они бы вообще столкнулись, как в гипотезе с Тейей.
Аноним 22/03/19 Птн 12:44:39 47228755
Лучи света же не исчезают? Т.к. я могу увидеть свет отдалённых звёзд.
Но куда девается часть, а всё понял.
Аноним 22/03/19 Птн 16:37:31 47235556
Кто-нибудь выходил в открытый космос на орбите Луны?
Аноним 22/03/19 Птн 16:45:12 47235957
>>472355
Только на самой Луне. А так все выходы были на низкой околоземной. Кстати совковая миссия на Луну предусматривала внешний переход между кораблями.
Аноним 22/03/19 Птн 16:52:49 47236458
>>472359
Да и светлое будущее она тоже предусматривала.
Аноним 22/03/19 Птн 17:37:04 47237559
>>471955
Разрушители мифов воспроизводили этот эксперимент - запустили Бастера на стуле с ракетами
Аноним 22/03/19 Птн 18:18:49 47239160
>>471955
Я помню, но не помню точного названия. Хотел бы взглянуть сейчас на эту энциклопедию, там столько интересных изображений было.
Аноним 22/03/19 Птн 18:28:14 47239561
20180307evans-e[...].jpg (1251Кб, 2200x2012)
2200x2012
>>472359
>>472355

На самой орбите Луны выходов не было, но астронавты Аполлонов 15, 16 и 17 после отлета к Земле вылезали наружу, чтобы собрать пленку с внешних камер. Даже фоточки с нашей планетой есть.
Аноним 22/03/19 Птн 19:41:35 47240162
Возможно странный вопрос, но: допустим космонавт застрял на орбите и съел всю еду. Как долго он сможет прожить высирая говно и поедая его?
Аноним 22/03/19 Птн 20:19:43 47240963
>>472401
Также долго, как просто сычуя на орбите без поедания говна - оно не питательное.
Но можно есть части себя. Такой рассказ был у Кинга.
Аноним 22/03/19 Птн 20:20:29 47241064
>>472401
Мама не рассказывала тебе что говно токсично?
Аноним 22/03/19 Птн 20:22:25 47241165
>>472410
>>472409
Я просто вспомнил пса, который сидел на привязи и ел своё говно.
Т.е. если съел всю еду, и рядом куча говна - не выживешь? Почему пёс может есть говно, а человек нет?
Аноним 22/03/19 Птн 20:57:05 47241966
Так, иду к вам из марсотреда. Можно ли передавать данные с орбиты Марса на Землю с высокой скоростью, скажем, >1 МБит/с, с одного орбитера (понятно, что с сотни орбитеров суммарно можно)? Если да, то почему это еще не сделано? Если нет, то что препятствует?
Аноним 22/03/19 Птн 21:00:39 47242067
>>472419
Очевидная масса/размер антенн и электрооборудования, сомнительная целесообразность
Аноним 22/03/19 Птн 21:04:02 47242268
>>472420
Целесообразность очень даже есть - и фоток гораздо больше сделать можно, и главное - видосики-видосики для черни, чтоб ее впечатлить. Вот я щас поискал видосики с Марса на ютубе - нихрена нет, пичаль.
Аноним 22/03/19 Птн 21:06:19 47242369
>>472420
Алсо, ядерный реактор нельзя запихнуть в орбитер для очень мощного источника энергии? Или типа источника питания кьюриосити, забыл как его?
Аноним 22/03/19 Птн 21:12:28 47242570
>>472422
Фоток и так хватает, во всяком случае я не слышал чтобы ученые жаловались. А видео мало что даст. В любом случае, увеличение массы и сложности увеличат стоимость настолько, что чуть больше финансирования из-за одобрения плебсом не покроют цену.
>>472423
На данный момент нет, только в разработке. Даже если сделают, все равно нет смысла ставить на аппараты, так как это будет неоправданное увеличение сложности, цены и массы. РИТЭГи очень дорогие так как делаются из особого плутония, и дают относительно мало энергии.
Аноним 22/03/19 Птн 21:18:52 47242771
>>472425
А развитие антенн и всякой такой херни, связанной с домашними роутерами, должно жеж ведь помочь в рамках существующих ограничений повысить скорость, если применить опыт разработчиков роутеров? Скорости роутеров-то очень сильно выросли за последние 15 лет
Аноним 22/03/19 Птн 21:23:17 47242872
>>472427
Это разное совсем. Для роутеров куча факторов имеет значение, в том числе программных. А на космических расстояниях все просто: диаметр передатчика - длина волны - диаметр приемника, только это имеет значение и это никак не наебешь.
Аноним 22/03/19 Птн 21:28:34 47243073
Аноним 22/03/19 Птн 21:46:04 47243374
MROrate.jpg (230Кб, 1918x944)
1918x944
>>472419
> Можно ли передавать данные с орбиты Марса на Землю с высокой скоростью, скажем, >1 МБит/с
Пикрелейтед.
Аноним 22/03/19 Птн 22:28:17 47243575
Предположим, что большую часть ядра Земли магическим образом заменили на золото, путем телепортации. У золота плотность гораздо выше, так что новое ядро будет тяжелее. Подгадали так, что ускорение свободного падения на поверхности выросло на 20%. С этого момента магия закончилась, дальше все работает строго по физическим законам.

Будут ли какие-нибудь значительные последствия, или только рухнет сотня ветхих сараев, да тысчонка старушек переломает ноги? Или будут неебические землетрясения, обвалы горных хребтов, вот это все?
Аноним 22/03/19 Птн 22:32:48 47243676
>>472435
Ну да, очевидно, самые высокие горы обрушатся. Ещё самые высокие деревья вымрут.
Аноним 22/03/19 Птн 22:53:22 47243877
>>471981
только приливы будут сильнее
Аноним 22/03/19 Птн 22:56:41 47243978
Аноним 22/03/19 Птн 22:58:36 47244079
image.png (456Кб, 569x869)
569x869
image.png (283Кб, 558x851)
558x851
Помогите с орбитой в пространстве.
Я нифига понять не могу вывод координат x, y, z в Декартовой системе координат при двух известных углах, радиусе сферы, а также времени. Нашёл пикрил, но с какого-то момента для меня это начинает выглядеть как "Рисуем сову".

Использование углов Эйлера не подходит, т.к. нет коммутативности, есть ещё кватернионы, но я даже не могу представить, что мне с ними делать.

Если проще, то интересует движение по окружности большого круга сферы.
Аноним 22/03/19 Птн 23:01:55 47244280
>>472433
https://marsmobile.jpl.nasa.gov/mro/mission/communications/
> The data rate is about 0.5 to 4 megabits per second
О, да, всё очень неплохо, я видать с ровером на Марсе попутал, это у него даялаповская скорость видать. Значит могли бы и видосики уже щас снимать в 4К
Аноним 23/03/19 Суб 21:36:10 47257981
>>472442
Лазерные коммуникации станут геймчейнджером.
Аноним 23/03/19 Суб 21:38:04 47258082
>>472579
Говорят хер точно направишь лузер на таком расстоянии
Аноним 23/03/19 Суб 21:40:46 47258183
Аноним 23/03/19 Суб 21:52:48 47258384
Аноним 23/03/19 Суб 21:55:28 47258485
>>472442
О, свидетельства разнятся, тут >>472583 уже 5,2
> Even at its maximum data rate of 5.2 megabits per second (Mbps), the Mars Reconnaissance Orbiter (MRO) requires 7.5 hours to transmit all of its onboard recorder,
Аноним 23/03/19 Суб 23:05:01 47260686
>>472583
Какое HD в дальнем космосе? Даже с лазорами там будут сотни килобит максимум
Аноним 24/03/19 Вск 00:03:28 47261887
>>472606
Тогда нафиг они нужны? Сотни килобит и даже 4 мбита/с и так уже есть
Аноним 24/03/19 Вск 09:33:21 47264388
145510277218635[...].jpg (273Кб, 1600x900)
1600x900
объясните тупому, если вселенная постоянно расширяется, каким образом сталкиваются галактики? они же удаляются друг от друга?
не понимаю
Аноним 24/03/19 Вск 09:56:12 47264489
Почему аквариумы делают из 4 стёкол скреплённых резиной, а не из цельной коробки из стекла?
Аноним 24/03/19 Вск 10:01:23 47264590
>>472643
Вселенная расширяется особым образом. Она не раздувается как шарик,но в ней постоянно появляется новое пространство, сейчас примерно 70 км/с на мегапарсек, то есть каждую секунду расстояние в мегапарсек увеличивается на 70 км и это число все время увеличивается, вселенная-то с ускорением расширяется. Но если галактики расположены на таком расстоянии друг от друга, чтобы притягивались сильнее, чем их разводит пространство, то расширение вселенной не будет успевать за ускоряющимся сближением галактик. Собсна, наш родной Млечный Путь и Андромеду в 2,5 млн св. лет через пару-тройку миллиардов лет ожидает такая же участь, как и М51.
Аноним 24/03/19 Вск 10:03:46 47264691
>>472645
Т.е. угасание подтверждено?
Аноним 24/03/19 Вск 10:10:24 47264792
Аноним 24/03/19 Вск 10:16:09 47264893
>>472647
Ну я читал что есть варианты развития вселенной: сжатие и угасание.
Аноним 24/03/19 Вск 10:21:25 47264994
>>472648
ЕМНИП, Большой Разрыв - один из вариантов того, как эта Вселенная ВСЁ. Есть такая штука, сфера Хаббла называется. Это область пространства, объекты в которой могут быть увидены наблюдателем в центре этой сферы. Объекты за ее пределом удаляются быстрее скорости света, а потому не могут быть увидены. С учетом ускоряющегося расширения, сфера Хаббла становится все меньше и меньше, если экстраполировать - то и до субатомных размеров может дойти. А в таком случае будет исключено взаимодействие между элементарными частицами, так как фотоны и глюоны перемещаются со скоростью света.
Аноним 24/03/19 Вск 10:29:24 47265095
>>472644
Бамп вопросу, а то у меня авкариум сегодня треснул, и я подумал...
Аноним 24/03/19 Вск 10:52:45 47265396
>>472644
Чтобы твари, живущие в нём, в попытках вырваться на волю разбили первое стекло, разбили второе стекло, уже не так решительно разбили третье, увидели бы, что ничего не меняется, осознали тщетность бытия и прекратили попытки покинуть их ограниченную ойкумену.
Аноним 24/03/19 Вск 11:20:51 47265497
DsLKwonxf8A.jpg (156Кб, 1080x1080)
1080x1080
Шо будет если в юпитер скинуть мощную водородную бомбу?
Аноним 24/03/19 Вск 11:28:15 47265698
>>472644
так проще плюс клей более эластичен
Аноним 24/03/19 Вск 11:30:53 47265799
>>472656
Так аквариуму эластичность не нужна. У меня просто прохудилась резинка в стенке и аквариум всё.
Аноним 24/03/19 Вск 11:31:09 472658100
>>472654
Бомба будет опускаться в атмосферу пока не сгорит или не раздавится давлением.
Аноним 24/03/19 Вск 11:31:53 472659101
Аноним 24/03/19 Вск 11:53:54 472661102
>>472658
>>472659
Имеется ввиду взорвать в атмосфере юпитера.
Аноним 24/03/19 Вск 11:57:05 472662103
>>472661
Конкретизируй.
Запустить термоядерную реакцию в атмосфере точно не выйдет если ты об этом.
Аноним 24/03/19 Вск 13:54:52 472673104
А были опытные или серийные бипланы, трипланы, многопланы с реактивными двигателями? И вообще, имеет ли многоплан право на существование сегодня или одно крыло всегда лучше?
Аноним 24/03/19 Вск 13:55:21 472674105
>>472661
Ничего не будет, водород-1 вообще не взрывается.
Аноним 24/03/19 Вск 14:10:28 472678106
ipgjjtyohjt11.jpg (119Кб, 1280x859)
1280x859
>>472673
M-15 Belphegor единственный серийный.

Бипланы полностью сдохли как класс еще до Второй Мировой, когда научились нормально считать аэродинамику, и выяснилось, что два крыла, расположенных друг над другом увеличивают подъемную силу меньше, чем в два раза, а аэродинамическое сопротивление — гораздо больше, чем в два.

Буквально единственное их преимущество это возможность хранить самолет в меньшем по размеру сарае, из-за чего в любительской авиации их до сих пор немножко осталось. Помимо этого, одно крыло лучше абсолютно всегда.
Аноним 24/03/19 Вск 14:47:31 472685107
001.jpg (8Кб, 354x142)
354x142
>>472673
Самый первый формально реактивный - Coandă-1910. Ещё задолго до турбин, предсжатие мотокомпрессором.
Аноним 24/03/19 Вск 15:17:27 472694108
>>472673
>И вообще, имеет ли многоплан право на существование сегодня или одно крыло всегда лучше?
В определённых нишах, где нужен неприхотливый слоупочный самолёт-маршрутка. Кукурузник Ан-2/Ан-3 в модернизированном виде до сих пор выпускается. Скорость сваливания у него околонулевая, в штопор его вогнать почти невозможно, и вообще с ним легко справится даже пьяный узбек - и это при том, что взлётно-посадочные характеристики у кукурузника почти вертолётные. Ну а бороться за скорость и эаэродинамическое качество в случае с кукурузником не нужно, потому что это кукурузник, блять, а не бизнес-джет.
Аноним 24/03/19 Вск 15:19:19 472696109
>>472678
Еще конструкция прочнее, что важно, если самолет сделан из говна и палок. И несколько более стабилен в воздухе.
Аноним 24/03/19 Вск 16:09:51 472725110
>>472674
А в солнце шо горит?
Аноним 24/03/19 Вск 16:48:11 472744111
Аноним 24/03/19 Вск 16:49:47 472745112
Аноним 24/03/19 Вск 17:16:49 472748113
>>472725
Именно шо горит, а не взрывается. Протон-протонный цикл вообще очень тухло идет, даже при охуительных температурах и давлениях солнечного ядра атому водорода нужно в среднем миллиард лет, чтобы прореагировать с другим атомом и превратиться в дейтерий, и дальше быстро-быстро собраться в гелий.

Ускорить процесс особо не выйдет, он ограничен не внешними параметрами, а вероятностью позитронного распада дипротона, которая околонулевая. Как сильно не сжимай и не нагревай, быстрее дейтерий из дипротона понаделать не получится, его распад полностью случаен.

Даже в самых массивных и ярких звездах протон-протонный цикл особо не идет, там другие процессы доминируют, но на Юпитере для них не те условия.
Аноним 24/03/19 Вск 17:25:15 472751114
alice-hamel-ast[...].jpg (497Кб, 1299x2000)
1299x2000
наверное платиновый вопрос, но всееж : из чего и как рождаются новые звезды, и что будет когда ресурс иссякнет? Новых звезд больше не будет?
Аноним 24/03/19 Вск 17:52:27 472756115
>>472748
В центре термоядерного взрыва температура и давление больше, чем в ядре солнца.
Аноним 24/03/19 Вск 17:54:41 472757116
>>472751
Т Е П Л О В А Я С М Е Р Т Ь В С Е Л Е Н Н О Й
Т С В
Е М С
П Е Е
Л Р Л
О Т Е
В Ь Н
А Н
Я О
Й
Аноним 24/03/19 Вск 18:21:54 472761117
>>472756
И долго она у тебя там больше?
Аноним 24/03/19 Вск 18:28:13 472763118
>>472756
Ну так реакция-то все равно не идет. Даже если вообще весь водород столкнулся и связался в дипротоны, энерговыделения нет, для энерговыделения нужен самопроизвольный позитронный распад в дейтерий.
Аноним 24/03/19 Вск 18:50:13 472765119
.
Аноним 24/03/19 Вск 18:50:20 472766120
>>472654
>Шо будет если в юпитер скинуть мощную водородную бомбу?
>Имеется ввиду взорвать в атмосфере юпитера.
Затухающая термоядерная реакция. Насколько затухающая - зависит от глубины взрыва.

Длительность зависит от овердохуя факторов, но с ростом мощности взрыва (вспышка царь-бомбы, например, длилась дольше минуты), и с ростом внешнего давления эта длительность будет расти. Алсо, в Юпитере ещё и дейтерия дохуя, и по некоторым теориям дейтерий там вполне тлеет.

Вероятно, царь-бомба в глубинах Юпитера породит термоядерную вспышку длительностью от нескольких дней до нескольких месяцев. Юпитер не распидорасит, да и размеры этой вспышки по сравнению с Юпитером будут довольно маленькими. Но на какой-то период его светимость заметно так возрастёт.
Аноним 24/03/19 Вск 18:52:38 472769121
Алсо, вот телега о D-D и D-T реакциях в недрах Юпитера, который по этой теории оказывается эдакой дейтериевой недозвездой.
https://iopscience.iop.org/article/10.1086/305797
Аноним 24/03/19 Вск 19:01:01 472772122
А в чем прекол дейтерия? Типа у него импульс больше, чем у протия?
Аноним 24/03/19 Вск 19:59:12 472787123
Господа, а вот чисто гипотетически есть у нас допустим охуевшая цивилизация в системе вроде нашей с населенной планетой вроде нашей. И вот допустим хотят эти охуевшие ребята больше лебенсраума себе в системе. И если для этой йоба цивилизации проблем с терраформированием уже нет - достаточно энергии, чтобы рай цветущий на орбите Плутона организовать, то гравитацию хочется как на родной планете. И вот этим гением, допустим, приходит идея начать сталкивать Европы с Амальтеями, чтобы собрать из кучи каменюк несколько спутников на орбите гигантов, соразмерных своей домашней планеты. И они начинают всякий мелкий песок из своего локального пояса Койпера, а то и в облако Оорта носятся за добавкой, собирать спутники, соразмерные земле на орбитах гигантов.
По условию задачи прицеплять йоба-движки можно ко всем объектам в системе, кроме родной планеты. Возможно ли собрать несколько шариков на орбитах гигантов, не подвергнув при этом родную планету вымиранию посредством полного распидораса траекторий в системе. Ну чисто теоретически? И можно ли организовать этим шарикам псевдостабильность на орбитах гигантов вне их охуевших радиоактивных поясов?
Аноним 24/03/19 Вск 20:22:57 472810124
>>472751
Звезды из водорода, но есть прикол, чем больше процент водорода в звезде, тем она живет меньше-выгорает буквально сотни миллионов лет и потом взрывается. Чем больше содержание других элементов, тем дольше живет звезда.
Ресурс водорода? Сейчас его 75% от массы всей вселенной. Процесса восстановления водорода пока не открыли, но возможно он существует, ведь как-то он образовывался в начале жизни вселенной.
Аноним 24/03/19 Вск 21:10:21 472838125
>>472772
У дейтерия глюонное облако жирненькое и сечение захвата сильно побольше. Это как не теннисным мячиком в мячик попадать, а нунчакой в нунчаку. Поэтому для самоподдерживающейся реакции требования к сжатию и температуре у него меньше на порядки.

>>472787
>По условию задачи прицеплять йоба-движки можно ко всем объектам в системе, кроме родной планеты. Возможно ли собрать несколько шариков на орбитах гигантов, не подвергнув при этом родную планету вымиранию посредством полного распидораса траекторий в системе. Ну чисто теоретически? И можно ли организовать этим шарикам псевдостабильность на орбитах гигантов вне их охуевших радиоактивных поясов?
Можно муравью хуй приделать, а ещё отсосать из родной звезды массу, сконвертировать в нужные элементы и построить сферу дайсона. В твоей задаче слишком много волшебства, поэтому и в ответе будет тоже много.
Аноним 24/03/19 Вск 21:17:05 472840126
>>472810
>Сейчас его 75% от массы всей вселенной.
Хуйню сказал, его около 10%. 80% массы вселенной - темная энергия и материя.
Аноним 24/03/19 Вск 21:21:27 472844127
>>472840
А ты там уже открыл что это не темный водород или просто выебываешься?
Аноним 24/03/19 Вск 21:22:02 472845128
>>472766
> в Юпитере ещё и дейтерия дохуя
Так это же предпосылка для термоядерной реакции? Вторая звезда в системе не может получиться? Или всё же маловат для самостоятельной поддержки реакции даже с таким мощным запалом?
Аноним 24/03/19 Вск 21:22:27 472846129
>>472844
Просто выебываюсь.
Аноним 24/03/19 Вск 21:25:27 472850130
>>472645
>Млечный Путь и Андромеду в 2,5 млн св. лет через пару-тройку миллиардов лет ожидает такая же участь, как и М51.
Блин, я не хочу. Надеюсь, человечество найдёт способ избежать катастрофы.
Аноним 24/03/19 Вск 21:25:35 472851131
>>472838
У дейтерия атомная масса 2,014, значит 2 атома = 4,028
У гелия 4,003
Это значит 0.025 электронами пренебрегаем переходит в энергию?
Аноним 24/03/19 Вск 21:27:14 472855132
>>472850
Не будет никакой катастрофы, слишком большое расстояние между звездами. Ну может пидорнет немного гамма-излучением.
Аноним 24/03/19 Вск 21:41:35 472871133
https://habr.com/ru/post/371363/

Собственно у меня вопрос, а что по излучению Хоккинга? Ведь эта черная дыра размером в 2 световых года должна выделять просто огромное его количество, которое наверняка можно как-то зафиксировать.
Аноним 24/03/19 Вск 21:41:58 472872134
Где в инете оргиналы/обработанные панорамы Венеры? Буду искать ящериц
Аноним 24/03/19 Вск 22:14:27 472885135
>>472871
Наоборот, чем массивнее черная дыра, тем меньше она излучает, и наоборот — по мере испарения и уменьшения ЧД излучение должно нарастать и в самом конце заканчиваться нихуевым взрывом.

Впрочем, даже черные дыры звездных масс излучают на порядки меньше, чем в них падает реликтового излучения, а процессы, порождающие менее массивные ЧД, науке не известны.
Аноним 24/03/19 Вск 22:14:45 472886136
Есть ли несложный способ, чтобы объект оставлял при дебаге отметки своего пути на экране в виде точек или линии?
Аноним 24/03/19 Вск 22:15:09 472887137
>>472886
Промахнулся с тредом.
Аноним 24/03/19 Вск 22:17:40 472888138
Кря-кря, зачем я здесь?
Аноним 24/03/19 Вск 22:21:12 472889139
Почаны, вы можете мне более-менее вменяемо объяснить, почему НАСА уже 50 с лишним лет не может в автоматическую блять стыковку?
Эта страна первой создала юзабельные сверхпроводники - транзисторы, запилила первые интегральные схемы, всегда имела мощнейшие в мире компьютеры (с начала 90х началась чехорда с азиатами, один год японский/китайский комп самый мощный, другой - штатовский). И эта страна не может в автоматизацию стыковки. Пиздец блять. Сука, что за хуета, как такое может быть. СССР уже с 67 года умеет в автостыковку - эти блядь в 2019 году пользуются рукой-крюкой на МКС. Ну не пиздец ли?
Я блядь не троллирую и не хочу разжечь нацсрач, я просто реально хочу понять, почему НАСА со своими технологиями на всех своих миссиях использует ручную стыковку. Почему, СУКА ПОЧЕМУ? Я знаю, что некоторые тут говорят "ИМ ПРОСТА НЕНУЖНО". Как блять не нужно? Это же удобно. Это же исключает в лишний раз не нужное (вот уж реально НЕНУЖНО) воздействие человеческого фактора.
Анончи, объясните мне, в чем тут прикол. Я из-за этого вопроса спать по ночам не могу.
Аноним 24/03/19 Вск 22:23:52 472890140
>>472889
Ты че поехавший? Как по твоему стыкуются цигнусы и драконы?
Аноним 24/03/19 Вск 22:32:37 472891141
>>472889
В абрамсах тоже ручная перезарядка пушки.
Аноним 24/03/19 Вск 22:33:26 472892142
>>472885
>а процессы, порождающие менее массивные ЧД, науке не известны.
БАК?
Почему, кстати, микроЧД после столкновения протонов, не взрываются хокинговским излучением?
Аноним 24/03/19 Вск 22:33:38 472893143
>>472618
Ты не путай Марс и дальний космос. С лазорами с Марса у тебя будет 11 мбит вместо 4. А из дип спейса - 300 кбит вместо 11.
А поколение ютуба хочет HD MARS PR0N ONLINE
Аноним 24/03/19 Вск 22:37:25 472895144
>>472892
Они настолько малы что может и взрываются, но незаметно
Аноним 24/03/19 Вск 22:43:00 472898145
Объясните, допустим я окажусь рядом с магнетаром, достаточно далеко чтобы меня не распидорасило гравитацией, но достаточно близко чтобы оказаться в магнитосфере.
Что именно со мной произойдет? Ну то есть понятно, что распидорасит, но каким образом? Все атомы в моем теле поляризует или что?
Аноним 24/03/19 Вск 22:47:57 472902146
>>472895
Планковская масса равна примерно 2 10^-8 кг. Ей соответствует энергия 2 * 10^-8 x (3 10^8)^2 = 1,8 10^9 Дж. Это почти полутонна в тротиловом эквиваленте.
Аноним 24/03/19 Вск 22:53:10 472904147
1024px-COTS2Dra[...].jpg (114Кб, 1024x852)
1024x852
>>472890
Раз ты не знаешь, то ты охуеешь.
Они стыкуются при помощи руки-крюки. Она используется для стыковки к МКС всех НЕроссийских аппаратов. Только последний Драгон, стыковавшийся в начале марта, делал это в автоматическом режиме без руки-крюки. Пиздец, а?
Аноним 24/03/19 Вск 23:19:21 472907148
>>472904
А, ты про конечный маневр, так бы сразу и сказал. Короче, это просто нинужно было.
Аноним 24/03/19 Вск 23:23:27 472910149
>>472889>>472904
Не, ну теперь-то можно говорить, что наконец-таки смогли.
ATV стыковался к российскому сегменту, соответственно, автоматически.
Dragon, Cygnus и HTV запилены под соединение со станцией с помощью CBM. Для автоматических кораблей такой вариант имеет свои плюсы.
Шаттл же... ну, а зачем же тогда нужен Space Shuttle Commander?
Аноним 24/03/19 Вск 23:34:28 472915150
>>472889
Значит с рукой было проще.
Теперь будут в авто режиме стыковаться, по крайней мере драконы пилотируемые. Напиши в наса письмо с этим вот твоим вопросом.
Аноним 24/03/19 Вск 23:36:26 472916151
>>472898
Все атомы железа вытянет из твоей жалкой тушки магнитным полем. Клетки крови разрушатся и внутренние органы тоже, и ты подохнешь.
Аноним 24/03/19 Вск 23:38:13 472917152
>>472902
Ну и как, часто сталкиваются протоны с такой энергией, чтобы ЧД образовать?
Аноним 24/03/19 Вск 23:38:40 472918153
рс17.jpg (181Кб, 600x600)
600x600
>>472751
Правильный арт, в космос лучше выходить в валенках.
Аноним 24/03/19 Вск 23:39:55 472919154
>>472916
То есть магнитное поле действует только на железо? почему?
Аноним 24/03/19 Вск 23:43:58 472920155
>>472917
Нечасто. Во всяких коллайдерах - вообще никогда, энергии не те. Просто это вообще минимально возможная масса для ЧД.
Аноним 25/03/19 Пнд 00:55:03 472924156
>>472872
Вроде как пара говенного кач-ва фоточек - это все что есть
Аноним 25/03/19 Пнд 00:55:47 472925157
Аноним 25/03/19 Пнд 01:01:35 472926158
>>472893
> С лазорами с Марса у тебя будет 11 мбит вместо 4
Ну это значит сей оратор >>472606 поднаврал? Потому что в описании хреновины, которую к Марсу запустят с лазерной коммуникацией, я тоже про сотни кБит/с читал.

В оптоволоконном SPDIF кабеле кста 1,5 мБит/с (битрейт АудиоСиДи), дак это кабель и он всего пару метров от силы.
Аноним 25/03/19 Пнд 01:02:47 472927159
>>472893
> А поколение ютуба хочет HD MARS PR0N ONLINE
В начале этого года даже в 8К шмагли с МКС, на ютубе есть
Аноним 25/03/19 Пнд 01:04:19 472928160
>>472919
На феромагнетики, наверно.
Кстати, если из тебя действительно железо вытащит, то если ты не погибнешь от полученных ранений, вызванных вылетом железа из твоего тела, то ты задохнёшься.
Аноним 25/03/19 Пнд 01:47:16 472929161
.
Аноним 25/03/19 Пнд 03:32:41 472931162
>>472851
>Это значит 0.025 переходит в энергию?
Именно. Так называемый дефект массы.

>>472892>>472902
Планковская ЧД испарится за планковское время. Чтобы она просуществовала хоть сколь-либо доступный для регистрации период, энергия должна быть на много-много порядков больше.

>>472898
>Что именно со мной произойдет?
По телу пойдёт напруга в хуиллиарды гигавольт и испарит тебя вихревыми токами.
Аноним 25/03/19 Пнд 09:08:56 472957163
Аноним 25/03/19 Пнд 10:04:14 472963164
>>472898
Парамагнетики и диамагнетики ведут себя по-разному в градиенте магнитного поля. Одни втягиваются в область сильного поля, другие — выталкиваются из неё. Вот этими-то силами тебя и распидорасит. И да, кровь не является ферромагнетиком, а вихревые токи не появляются от магнитостатического поля. Но если поле вращается, то да, испарит вихревыми токами.
Аноним 25/03/19 Пнд 10:52:51 472978165
>>472889
>сверхпроводники
Полупроводники, валенок.
Аноним 25/03/19 Пнд 11:03:13 472983166
Аноним 25/03/19 Пнд 11:05:59 472985167
>>472917
Никак, есть предел энергий, при котором достаточно быстрая частица начинает эффективно тормозиться о реликтовое излучение. Этот предел на много порядков ниже планковской энергии/массы. Было много разговоров о якобы регистрации космических лучей с большей энергией, но в итоге существование предела было надежно установлено, а утверждающие обратное обосрались. Это может свидетельствовать в пользу принципиальной экспериментальной непроверяемости квантовых теорий гравитации, по крайней мере методами ускорительной физики.
Аноним 25/03/19 Пнд 12:14:45 473019168
>>472963
>Но если поле вращается, то да, испарит вихревыми токами.
Скорее всего вращается, ведь все звезды вращаются. Ну либо скорость вращения синхронизирована с орбитальным периодом
Аноним 25/03/19 Пнд 12:36:28 473026169
>>473019
Бля, да каждый раз когда черная дыра поглощает что-то массивное вроде звезды или другой черной дыры, она приобретает момент вращения. Стационарных невращающихся черных дыр наверное не существует просто по природе.
Аноним 25/03/19 Пнд 13:30:58 473054170
Можно ли использовать фотосферу Солнца как среду для накачки лазера?
Аноним 25/03/19 Пнд 13:38:59 473057171
>>473054
А солнечные батареи как по твоему работают? Подключаешь лазер к солнечным батареям и вперёд.
Аноним 25/03/19 Пнд 13:55:57 473066172
>>473057
я имею в виду два зеркала на орбите и луч проходит по касательной через фотосферу
Аноним 25/03/19 Пнд 13:56:50 473067173
Ахуенно жить на планете в пределах действия магнетара, где магнитное поле в районе 10-50 Тл. Для живых организмов безвредно и можно получать электроэнергию нахаляву просто из вращения планеты вокруг магнетара и самого магнетара.
Аноним 25/03/19 Пнд 13:59:28 473068174
>>473066
Лучше нарисуй, тут вообще не понятно, как ты представляешь себе работу лазеров и чего ты хочешь добиться.
Аноним 25/03/19 Пнд 14:56:05 473080175
Screen Shot 201[...].png (75Кб, 1061x675)
1061x675
Аноним 25/03/19 Пнд 15:00:49 473082176
>>473080
Сначала подумал "нихуя себе, даже гравитационное линзирование учтено", но потом понял, что у тебя просто кривые руки.

Теоретически можно, на практике требуется точность установки зеркал, сравнимая с длиной световой волны, как ты их на орбите с такой точностью повесишь?
Аноним 25/03/19 Пнд 15:09:38 473088177
>>473082
ну хуй знает, в космосе вот мазеры как-то сами получаются. может светить с одного аппарата на другой вспомогательным лучом и его использовать для наводки
Аноним 25/03/19 Пнд 15:54:10 473101178
lol3.png (7Кб, 231x176)
231x176
Аноним 25/03/19 Пнд 17:01:22 473112179
>>472931
>Планковская ЧД испарится за планковское время
Или будет стабильна и не подвержена хокинговскому испарению ваще, есть и такая гипотеза. В любом случае, смысл в том, что планковская масса - минимальная возможная для ЧД. Не заметить взрыв полутонны ТНТ в эквиваленте довольно сложно, так что "Они настолько малы что может и взрываются, но незаметно" выглядит сомнительно.
Аноним 25/03/19 Пнд 17:13:06 473115180
Почему наработки по Н-1 не были использованы при создании Энергии?
Аноним 25/03/19 Пнд 17:31:13 473118181
>>473115
Там особо и нечего было для Энергии использовать. Мощные водородники с нуля делали, боковушки рассчитывали на реюз и использование, как первая ступень для ракет более легкого класса ("Зенит", собсна, оно и есть). У Н-1 же была совершенно уебищная с точки зрения массового совершенства конструкция с подвесными шарообразными баками (хотя красивая, да) и движки, ЕМНИП, одноразовые, то есть вообще одноразовые. Их даже испытывали так: брали каждый третий движок из партии и прожигали на стенде. Робят - значит всю партию принимают. Пятый пуск должен был быть с нормальными НК-33 вместо НК-15, но к этому времени уже и звездно-полосатый флаг на Луне стоял, да и денег вбухали немеряно без особого выхлопа.
Аноним 25/03/19 Пнд 17:33:39 473119182
>>473115
Много наземной инфраструктуры использовалось
Аноним 25/03/19 Пнд 18:58:26 473141183
Пацаны, пацаны, поясните за автоматическую посадку Бурана. У нас тут конфликт военный возник на этой почве.
Я погуглил по доске, но ничего нормально не нашел. Вот этот вот молодой (я надеюсь, что вы еще молоды и полны сил) человек (я надеюсь, что не ошибаюсь) >>430892 упомянул, что прошлым летом в спейсаче подробно поясняли за автоматическую посадку Бурана.
Обращаюсь к этому молодому человеку, его друзьям, знающим людям и просто ко всем интересующимся - расскажите, пожалуйста, за эту посадку.
Была ли посадка вообще? В смысле, была ли она полностью автоматизирована от и до? Как отстающему в компьютерной технике СССР удалось сделать это? Если система хорошо сработала, то почему проект сразу же закрыли, а наработки по автоматическому взлету и посадке нигде больше не использовали? Ни у кого в мире ничего подобного просто не существует, а у вас полностью рабочая система - бери и пользуйся. Почему забили-то?
С уважением и комсомольским приветом, шаловливым поцелуем и ласковым игривым взглядом искренне ваш ученик 3 школы Петропавловска Сергей Ерохин
Аноним 25/03/19 Пнд 19:06:23 473143184
>>473141
>Была ли посадка вообще? В смысле, была ли она полностью автоматизирована от и до?
была
>Как отстающему в компьютерной технике СССР удалось сделать это?
оптимизация программ и алгоритмов под конкретную полосу
>Если система хорошо сработала, то почему проект сразу же закрыли, а наработки по автоматическому взлету и посадке нигде больше не использовали?
сделать способный сесть где угодно автопилот намного сложнее, а без этого нужность на обычных самолетах сомнительна
>Ни у кого в мире ничего подобного просто не существует, а у вас полностью рабочая система
тот же шатол мог, но не делал чтобы пилоты не чувствовали себя пассажирами
Аноним 25/03/19 Пнд 20:19:34 473158185
Аноним 25/03/19 Пнд 20:50:15 473165186
>>473158
Ну погугли, в википедии в статье про сссровский апеарат эти фотачки есть вроде
Аноним 25/03/19 Пнд 20:51:00 473166187
>>473165
Аппарат, который лэндер на Венеру был, Венера-5 штоле или как-то так
Аноним 25/03/19 Пнд 20:56:02 473167188
>>473141
>Как отстающему в компьютерной технике СССР удалось сделать это?
Отставание в элементной базе != отставание в методологии разработки. Плюс лазерный фокус на конкретном решении. Примерно как и с Р-7 было, и с многим другим. Ну и в общем-то ничего принципиально нового такая автономная система принятия решений (единственное чем отличался автопилот бурана от шаттла) не представляла, такие в штатах давно были, но в таком К Р А С U В О М с точки зрения обывателя виде не применялись. Тому же шаттлу это нахуй не упало, ибо обезьянки всегда на борту. (что само по себе было объектом критики задолго до первого полёта, и поделом).
>почему проект сразу же закрыли, а наработки по автоматическому взлету и посадке нигде больше не использовали?
Закрыли потому что совок наебнулся и денег не было. Наработки по конкретному аппарату не использовали потому что они были специфичны, как заметил анон выше. А ноу-хау вполне использовали, ИСС например адаптировали под свои нужды Дракон, методологию, общие принципы, и с годами превратили это в кастомное инхаус решение вперемешку с Ada. Плюс разделили спеки и реализацию бортового софта по изолированным отделам по образцу программы Э-Б (где этим вообще разные организации занимались). В ИСС назвали это "отказались от программистов", что конечно же не так. Получился такой доморощенный франкенштейн-конвеер, которым они вроде бы довольны и обучают ему приходящих специалистов. Не знаю насколько это эффективно на практике, с дивана пиздеть не буду.

По тому срачу скажу что оба диваны, бураноёб не знает банальных вещей про ОС-120 и историю создания комплекса, крыльехейтер слишком хейтит крылья на основе ЯСКОЗАЛ и ложных аналогий.
Аноним 25/03/19 Пнд 21:12:27 473169189
>>473167
> ноу-хау вполне использовали, ИСС например адаптировали под свои нужды Дракон, методологию, общие принципы, и с годами превратили это в кастомное инхаус решение вперемешку с Ada. Плюс разделили спеки и реализацию бортового софта по изолированным отделам по образцу программы Э-Б (где этим вообще разные организации занимались). В ИСС назвали это "отказались от программистов", что конечно же не так. Получился такой доморощенный франкенштейн-конвеер, которым они вроде бы довольны и обучают ему приходящих специалистов.
Чего блять? Я не знаю, за что вы тут сретесь, просто мимо прочитал и охуел от перегрузки терминов. Какие блять нахуй ИСС, Ада, Э-Б и при чем здесь Дракон? Чего блять?
Аноним 25/03/19 Пнд 21:20:18 473172190
>>473169
>Э-Б
Энергия-Буран, дебил.
А за остальное я и сам не понял.
Аноним 25/03/19 Пнд 21:25:17 473177191
>>473169
ИСС = Решетнёвка (превая ссылка в гугле). Ada == язык программирования, наверное.

Дракон == язык программирования, запиленный под программу "Энергия-Буран". Под шаттл тоже свой запилили, HAL/S называется.
Аноним 25/03/19 Пнд 21:32:20 473180192
>>473177
>Дракон == язык программирования
Больно прогрессивный язык что ли какой-то? Аналогов нет?
Аноним 25/03/19 Пнд 21:33:16 473181193
>>473180
Никогда не слыхал
Погромист
Аноним 25/03/19 Пнд 21:36:13 473183194
>>473177
Щас бы под каждый пылесос пилить свой язык, вместо библиотеки нужных функций.
Аноним 25/03/19 Пнд 21:39:31 473186195
>>473180
Я для кого добавил про HAL/S, а?

Да, у космической области бывают специфические требования (в основном по надёжности). Нифига себе ВНЕЗАПНОСТЬ!
Аноним 25/03/19 Пнд 21:43:23 473189196
>>473180
Ну, такое. Просто техническое решение конкретного коллектива под специфические требования. Обычный визуальный язык программирования.

>>473169
>Какие блять нахуй ИСС, Ада
Бля, ну я понимаю что тред тупых вопросов, но хоть погуглить не мешало бы. Ada - язык, часто используемый в высоконадежных системах. Кроме него, в ИСС Модулу-2 юзают (по крайней мере лет 5-7 назад точно юзали, вряд ли что-то изменилось). ИСС - основной российский производитель спутников.
Аноним 25/03/19 Пнд 21:47:19 473190197
l.jpg (75Кб, 700x460)
700x460
>>473167
Дружелюбный
Русский
Алгоритмический язык,
Который
Обеспечивает
Наглядность
Аноним 25/03/19 Пнд 21:49:39 473191198
>>473189
А я думал ИСС это ISS
Аноним 25/03/19 Пнд 21:51:50 473193199
>>473190
Типично для позднего совка, лол. А так-то аббревиатур подобных везде полно, а автор Дракона был типичный академический евангелист, носящийся с охуительной идеей, коих полно в мировом компьютер саенсе, начиная от визуального программирования и кончая высоколобым ФП вроде хаскелля.
Аноним 25/03/19 Пнд 22:06:18 473198200
>>473180
Ну, один из ранних визуальных языков программирования (где кодишь через всякие блок-схемы хуе-мое). Массовому юзеру из-за недоступности был не нужен, в то время уже родились всякие всемирнопопулярные Матлабы - хуета из той же оперы. Как Совок приказал долго жить, то его попробовали перепилить под гражданские нужды, создав свободные доступные простому смертном компиляторы под Винду, Линукс и прочие. Не взлетело и вряд ли кто-то, кроме ИССовцев им пользуется.
Аноним 25/03/19 Пнд 22:15:11 473203201
>>473198
>вряд ли кто-то, кроме ИССовцев им пользуется
На самом деле, практически весь крупный российский аэрокосмос, который занимается большими изделиями. ИСС это лишь пример наиболее крупного внедрения. Так-то на основе дракона запилили первую отечественную CASE, которую внедрили как минимум в лавке, энергии, прогрессе, хрюничеве и тп. С тех пор оно эволюционировало.

Да, специфическое решение для высоконадежных систем, которых так-то целая пачка имеется.
Аноним 25/03/19 Пнд 22:40:15 473213202
>>472927
Потому что всего 400км. 8К на таком расстоянии можно получить даже почтовыми голубями. В скафандрах. Ну или Союзом, набитым флешками.
Аноним 25/03/19 Пнд 22:41:36 473215203
>>472926
Хреновина действительно умеет только одну моду и 330кбит. Ее оттестируют в дипспейсе, а у Марса развернут станцию, которая сможет в многомодовую передачу.
Аноним 26/03/19 Втр 11:01:12 473274204
Аноним 26/03/19 Втр 13:58:00 473298205
>>473203
Че-т походу какой-то говнистый у них язычок. Программирования.
В смысле, почему отечественные спутники (как в Совковые, так и в нынешние) так сильно уступают в продолжительности жизни амеровских спутникам? Наши блять за год сдыхают и они все новые и новые им на замену постоянно отправляют. Год полетал - новый, год полетал - новый. А американские по 30 лет на орбите рабочие висят.
Что-то походу кодить блок схемами было хуевой идей.
Аноним 26/03/19 Втр 14:09:41 473300206
>>473298
Потому что в Россию запрещено поставлять электронику класса aerospace, чтобы она не досталась военным, и спутники приходится клепать на незащищённой электронике, которую быстрее убивает космическое излучение?
Аноним 26/03/19 Втр 14:12:13 473301207
>>473300
А почему у нас этого не придумали?
Аноним 26/03/19 Втр 14:28:39 473302208
Зачем космонавты в скафандрах с закрытыми забралами, ведь в челноках есть кислород.
Аноним 26/03/19 Втр 14:40:45 473304209
>>473300
Я слышал что в России производят электронику такого класса, но очень мало. Почему производство не увеличить, если это такая йоба которую все хотят, и не продавать в Европу?
Аноним 26/03/19 Втр 14:42:30 473305210
>>473302
зачем ты неправильно задаешь вопросы?
Аноним 26/03/19 Втр 14:43:03 473306211
>>473305
А как задавать, я не понял...
Аноним 26/03/19 Втр 14:44:41 473307212
Аноним 26/03/19 Втр 14:44:53 473308213
>>473298
Каким боком язык программирования к САС вообще? Это вроде тред тупых вопросов, а не ответов.
>Год полетал - новый, год полетал - новый
Это ты с честным лицом порешь чушь, или на срач напрашиваешься? Ну на примере того же Глонасс-М, гарантированный САС 7 лет, по факту в составе группировки сейчас четыре 12-летних аппарата. Это орбита 19 тыс км, внешний радиационный пояс, высокоэнергетичный. Это конечно далеко не USA-96, которому 25 лет, но 1) эти аппараты первой очереди и ещё сколько-то отслужат, 2) USA-96 на ладан дышит, 2.5 года был выведен из строя из-за глючной памяти, и 3) все его соседи по группировке давно вышли из строя, это ошибка выжившего.

>>473304
Радхард электроника никому не нужна, на самом деле. Спрос на неё поштучный, а массовая ПП индустрия оперирует семизначными суммами. Те три конторы что делают радхард в штатах , делают это за бешеный прайс и всё равно дышат на ладан, ибо крайняя мелкосерийность не только в производстве, но и в разработке.
Аноним 26/03/19 Втр 14:45:04 473309214
>>473306
как забрала скафандров, челноки и кислород вообще связаны.
Аноним 26/03/19 Втр 14:47:10 473311215
>>473308
> семизначными тиражами
тогда уж
Аноним 26/03/19 Втр 14:48:05 473312216
>>473309
Ну, космонафт залезает в отсек, закрывает шлем и улетает. Я в терминологии не очень просто.
А зачем он закрывает шлем и дышит внутренним кислородом, а не там что снаружи?

>>473307
Ну не из-за этого же.
Аноним 26/03/19 Втр 14:49:49 473313217
>>473312
>Ну не из-за этого же.
Из-за этого, блять. На случай внезапной разгерметизации КК. ты какой-то совсем тупой
Аноним 26/03/19 Втр 14:51:11 473314218
>>473313
Стоп, т.е. все космонавты во всех странах начали летать в скафандрах из-за этого случая? Который вроде и так поняли почему произошёл?
А в самолётах зачем летают с кислородной маской всякие пилоты? Вроде в кабине кислород есть.
Аноним 26/03/19 Втр 14:55:04 473315219
>>473314
Тебе сказали - может произойти внезапная резкая или незаметная разгерметизация, или утечка технического газа, или пожар.
Аноним 26/03/19 Втр 14:55:13 473316220
>>473314
Кабины истребителей частично компенсируются по давлению с внешней средой, чтобы уменьшить нагрузку на фонарь.
На большой высоте дышать в кабине станет невозможно, поэтому лётчики дышат через маску.
Аноним 26/03/19 Втр 14:57:00 473317221
>>473316
А в танках или наземных юнитах нет такого чтобы юзали маски или подобное?
Аноним 26/03/19 Втр 14:58:02 473318222
>>473314
>Стоп, т.е. все космонавты во всех странах начали летать в скафандрах из-за этого случая?
Все космонавты во всех странах пока что летают только на одном типе кораблей, пока что других нет, после списания Шаттлов. (на Шаттлах тоже летали в высотных костюмах ACES, оранжевые такие - по той же причине, на случай разгерметизации) На новых, которые испытываются, тоже будут летать в высотных костюмах.

>Который вроде и так поняли почему произошёл?
Потому и летают. Вдруг произойдёт снова. Это дополнительный барьер.

>А в самолётах зачем летают с кислородной маской всякие пилоты?
Это в основном либо боевые пилоты, у которых нет нормальных условий в кабине, и всякое может произойти. Либо мелкая авиация, у которой кабина просто не герметизирована зачастую.
Аноним 26/03/19 Втр 15:01:27 473319223
>>473300
Радхард электроника здесь не при чем, и эмбарго не при чем (его не было некоторое время назад). Расчётные дозы-то одинаковые и у тех и у этих. Причина в том, что ну просто не умеют в большой САС, по куче разных причин. Это комплексная проблема, не надо все упрощать до "электроники".
Аноним 26/03/19 Втр 15:02:24 473320224
>>473317
Зачем?
В танках есть система очистки воздуха, чтобы экипаж не дышал пылью и боевыми отравляющими веществами, а больше и припомнить ничего подобного не могу.
Аноним 26/03/19 Втр 15:03:26 473321225
>>473308
А что насчет всяких воякских "Космосов"? Их только официально за 2.5к запущено. Хуй знает, сколько еще не разглашенных. Они же постоянно гробятся. Вот, недавно только новость была, что один из последних, проработав год (или меньше?), скоро наебнется.
Американские вояки так много вроде не запускают военных спутников.
>того же Глонасс-М, гарантированный САС 7 лет, по факту в составе группировки сейчас четыре 12-летних аппарата
Даже не будем упоминать, что юзабельным Глонасс стал только 9 лет назад (и то не совсем).
Аноним 26/03/19 Втр 15:06:04 473322226
>>473320
Прост я делаю концепт шагохода, и думаю на счёт костюма пилота. Вот думаю делать ему закрытый скафандр или без шлема тоже ок...
Аноним 26/03/19 Втр 15:15:40 473323227
>>473321
Не все "Космосы" военные. Там какая-то ебанутая система, по которой практически любой запуск могут занести в серию "Космос".
Аноним 26/03/19 Втр 15:15:57 473324228
>>473321
>А что насчет всяких воякских "Космосов"?
Это общий индекс для самых разных аппаратов, там чего только нет. В совке вообще все неудачные пуски по возможности называли космосом и делали вид что так и было задумано.

>Они же постоянно гробятся.
Ты прочитал в новостях != гробятся постоянно. Ты же не читаешь новости про те что работают годами по назначению. "Сегодня ничего не произошло", охуенная новость была бы. Нет ничего хуже, чем прочитавшие в новостях.

Тем не менее контролю качества пизда давно, тут базару нет, да и САС реально хуевый. Но не так, как ты это пишешь, лол. ИСС те же более-менее научились в САС средней паршивости.

>Даже не будем упоминать, что юзабельным Глонасс стал только 9 лет назад (и то не совсем).
Юзабельным он был еще в 1993 году, вот первые аппараты Глонасс были реально ломучее говно, и бабок на поддержание группировки не было. И что значит не совсем? Работает же все.
Аноним 26/03/19 Втр 17:18:31 473346229
Почему так сложно сделать компьютер, который бы работал на Венере? Просто нужно найти компоненты, которые бы не плавились и работали при более высокой температуре. Как можно было бы обычную жизнь улучшить, если бы у нас были несгораемые компьютеры и материалы.
Аноним 26/03/19 Втр 17:21:58 473348230
>>473346
Нахуя тебе компьютеры на Венере, ебан? Там ничего нет.
Это помойка похуже Марса и Меркурия. Туда ни один вменяемый человек прогать на ПМЖ полетит. Все в Таиланд едут, кодить, лежа у пляжа. Нахуя тебе на Венере компьютер?
Аноним 26/03/19 Втр 17:23:26 473350231
>>473348
Сделать там берлогу под землёй и жить, не?
Аноним 26/03/19 Втр 17:27:48 473351232
1.jpg (696Кб, 1861x965)
1861x965
>>473348
Наука жи есть. Венера крутая планета, не пзди мне тут. Особенно тамошние горы крутые. Там есть регион наподобие Гималаев с горами 11 км в высоту, который пробит кратером от астероида. Дыра в горах нафиг. Надо в горы садиться на Венере, там давление ниже должно быть. А наши советские аппараты в низины садились какие-то.
Аноним 26/03/19 Втр 17:31:29 473352233
>>473351
Кто такое пацанское картографирование Венеры сделал? Магеллан?
Аноним 26/03/19 Втр 17:34:54 473353234
>>473352
Гугел. Написано же.
Аноним 26/03/19 Втр 17:36:10 473354235
CVenera15New.jpg (64Кб, 600x434)
600x434
CVenera15Altime[...].jpg (40Кб, 620x620)
620x620
>>473352
Магеллан, Пионер и Венера-15/16, а теперь мы можем это на гугле смотреть
https://www.google.ru/maps/space/venus/

В Европе обещают аппарат с новым радаром, который вплоть до нескольких метров создаст новую карту. Но запустят не скоро.
Аноним 26/03/19 Втр 17:38:06 473355236
>>473354
>который вплоть до нескольких метров создаст новую карту
Пиздос, а когда землю с масштабом до нескольких хотя бы десятков метров зделают?
Аноним 26/03/19 Втр 17:39:02 473356237
Аноним 26/03/19 Втр 17:41:07 473357238
>>473354
Респект таким пацанам. У НАСЫ и ЕКА (особенно у НАСА) вообще охуенное картографирование планет.
Наверняка подфотошопливают, чтобы красивее смотрелось.
Аноним 26/03/19 Втр 17:41:15 473358239
>>473356
Где найти? Искал карты высот, нашёл только насовские карты с точностю около 100 или 500 метров на точку.
Аноним 26/03/19 Втр 17:45:15 473359240
>>473357
Японцы кстати тоже норм науки собрали на Венере недавно. Только поганый Роскосмос не может дальше НОО улететь.
Аноним 26/03/19 Втр 17:46:05 473360241
>>473357
>Наверняка подфотошопливают, чтобы красивее смотрелось.
Конечно.
Был бы у всех такой бюджет, как у НАСЫ, то тоже кучу крутых фоток бы делали.
Аноним 26/03/19 Втр 17:46:22 473361242
screamingcat.png (134Кб, 426x346)
426x346
>>473359
>Только поганый Роскосмос не может дальше НОО улететь.
Кто сидит в спейсаче в засмеялся тредах молчит.
Аноним 26/03/19 Втр 17:46:55 473362243
>>473360
Ага, с вирджинских островов.
Аноним 26/03/19 Втр 17:51:32 473364244
Насколько близко к Солнцу космический аппарат может крутиться продолжительное время? Читал про диэлектрические зеркала, пишут, что их можно изготовить так, чтобы по всему солнечному спектру отражаемость была >99%. Можно из них сделать большой теплозащитный щит?
Аноним 26/03/19 Втр 18:00:11 473365245
sentinel1antarc[...].jpg (10245Кб, 3473x5636)
3473x5636
>>473358
Я наверное напиздел. Полные радарные карты Земли сложно найти, может потому что они продают это говно как продукт, или потому что это непопулярное говно чтобы кто-то тратил усилия и создавал карты, которые широким слоям населения не будут интересны. Для Венеры это не роскошь, а необходимость, потому что кроме радара ничего нет, а для Земли радар это именно что роскошь, которая не всем нужна. Может поискать данные Сентинела-1 от ЕКА, там крутые снимки есть.
https://directory.eoportal.org/web/eoportal/satellite-missions/c-missions/copernicus-sentinel-1

Плюс НАСА и Индия сейчас вот эту няшу скоро запустят, но я тоже не знаю будет ли тут полное радарное картографирование
https://en.wikipedia.org/wiki/NISAR_(satellite)
Аноним 26/03/19 Втр 18:00:29 473366246
Аноним 26/03/19 Втр 18:07:48 473367247
Аноним 26/03/19 Втр 18:13:11 473368248
>>473367
>Сегодня 226 день из 2686 дней полёта до первого близкого приближения к Солнцу
Близкое приближение на скоростной скорости.
Аноним 26/03/19 Втр 18:17:00 473369249
>>473366
Лол, у него там большая половина постов в духе "Такую страну проебали, Америка уже не та, что раньше, все проебано".
Интеллектуалам везде все время не нравится.
Аноним 26/03/19 Втр 18:17:34 473370250
>>473369
Я слышал что сейчас в Америке жить очень хорошо.
Аноним 26/03/19 Втр 18:31:43 473371251
>>473368
>изкое приближение на скоростной скорости.
Это лучшее что можно сделать за деньги сейчас.
Аноним 26/03/19 Втр 18:32:44 473372252
>>473370
Ну в США возможно, а в какой-нибудь Колумбии экипаж STS-107 подтвердит или Венесуэле наверное так себе.
Аноним 26/03/19 Втр 18:34:59 473373253
>>473366

Всё правильно пишет, МКС нинужна. Лучше вместе ебанем по Европе.
Аноним 26/03/19 Втр 19:01:55 473376254
gallery37831832[...].jfif (19Кб, 480x306)
480x306
>>473373
>ебанем по Европе.
привет ты чё охуел
Аноним 26/03/19 Втр 19:09:08 473377255
изображение.png (1477Кб, 800x800)
800x800
изображение.png (457Кб, 1019x816)
1019x816
изображение.png (1315Кб, 768x610)
768x610
>>473373
А я и не против. Давно пора отправить наши боевые субмарины бороздить просторы Европы.
Аноним 26/03/19 Втр 19:11:17 473378256
>>473376
>пик
Это призыв устроить блокаду Европы?
Аноним 26/03/19 Втр 19:29:10 473379257
>>473378
Пиздец, в ту-кей-девятнадцать не уметь в американский лангуаге:
ВСЕ ЭТИ МИРЫ - ARE ТВОЁ, КРОМЕ EUROPA. ATTEMПT НИКАКОГО ВЫСАДКИ. НЕ ИСПОЛЬЗУЙТЕ. ИСПОЛЬЗУЙТЕ ИХ В ПОКОЙ.
Аноним 26/03/19 Втр 19:43:11 473380258
Вопрос по орбите. Если у нас корабль движется вокруг земли по кругу, но при этом сам имеет скорость, скажем, 120 км/с, а движение по кругу вокруг земли поддерживается вспомогательными двигателями (пусть топливо - не проблема), то вот при такой скорости на орбиту этого корабля масса планеты сильное влияние будет оказывать?
Аноним 26/03/19 Втр 19:48:53 473381259
>>473380
Как только ты вышел на орбиту тебе не нужно ничего поддерживать и разгонять, ты уже двигаешься с нужной скоростью чтобы оставаться на орбите. Масса планеты определяет отношение между твоей орбитальной скоростью и высотой. Если будешь скорость добавлять по направлению вращения - увеличишь высоту орбиты и вытянутость орбиты с одной стороны.
Аноним 26/03/19 Втр 19:50:02 473382260
>>473380
Ну как обычно, F=GMm/r^2. Если я правильно тебя понял, то вклад планеты в движение корабля будет зависеть от собственно массы планеты и от расстояния от неё.

Так-то при наличии волшебных двигателей и планета никакая не нужно, можно устроить орбиту вокруг пустого места.
Аноним 26/03/19 Втр 19:50:44 473383261
1551054228054.png (156Кб, 400x416)
400x416
>>473380
>120 км/с

Сука ты охуел блядь с такой скоростью около Земли летать, сбейте его кто-нибудь
Аноним 26/03/19 Втр 19:54:35 473384262
>>473383
Скажи спасибо, что я оттормозился по пути от солнца.
Аноним 26/03/19 Втр 19:57:44 473385263
>>473380
>120 км/с
Нифига ж себе.
Скорость, которой хватит, чтобы улететь нахуй из Солнечной системы - чуть больше 16,6 км/с, а у тебя 120.
При такой скорости ему вообще будет пофигу на чью-либо гравитацию, его путь через систему будет не орбитой, а почти идеальной прямой.
>движение по кругу вокруг земли поддерживается вспомогательными двигателями (пусть топливо - не проблема)
Если он летает всё-таки в космосе, а не в атмосфере, то особой помощи двигателей ему и не понадобится, скорость гасить-то нечему (ну, на самом деле есть тонкости, но вряд ли тут они имеются ввиду). Набрал и прёшь по инерции.
>то вот при такой скорости на орбиту этого корабля масса планеты сильное влияние будет оказывать?
>120 км/с
Если мы говорим о Земле - только если он попытается пройти через ядро планеты (с).
Тут проблема в другом: на такой скорости никакой орбиты не будет, тем паче круговой. Если только вокруг центра Галактики.
Аноним 26/03/19 Втр 20:04:21 473387264
>>473385
>>473381
Спейсаны, анон же прямым текстом сказал, что у него сайфайные движки поддерживают "орбитальность" движения. Т.е. они просто жарят в барицентр системы, имитируя таким образом гравитацию.
Аноним 26/03/19 Втр 20:06:44 473391265
>>473387
какая-то сцай файная хуйня, крутиться как укупник вокруг параши непонятно зачем. Если у тебя изначально хуйня сайфайная, то мы не можем знать как реальная гравитация будет влиять на полёт этой хуйни.
Аноним 26/03/19 Втр 21:01:58 473430266
изображение.png (687Кб, 672x378)
672x378
>>473141
>>473143
>>473167
Жомапель Харчок, бля.
https://www.youtube.com/watch?v=KI5DPQGGzaQ
Разрешите вторгнуться в вашу интеллектуальную беседу.
Скажу по секрету, я не очень верю в этот чудесный полет "Бурана". Но не спешите меня сразу же хуесосить и посылать в /zog/ач, мое мнение основано на объективных наблюдениях шизика.
Дело в том, друзья, что если этот аппарат действительно был настолько хорош, что способен взлетать, летать и садиться в полностью автономном режиме, то СССР создали, возможно, величайшую технологию всех времен и народов, самую грандиозную техническую разработку в истории человечества. Ни один современный беспилотник, созданный на новейшей элементной базе, не способен выполнить все эти этапы самостоятельно. Его ведет оператор. А вот "Буран" каким-то чудом мог. В 1988 году. 30 лет назад. 1 раз. И больше никто в истории ни разу не смог. Только "Буран". 30 блять лет назад.
Я знаю, что антоны скажут "Это просто надрачивание аппарата на взлет (который к тому же идет в связке с "Энергией", что делает этот этап не отличающимся разительно от запуска обычной ракеты-носителя с каким-либо грузом) и посадку на одну определенную полосу определенного космодрома, здесь нет ничего сверхнеобычного". Извините меня. Это охуеть как необычно. Все расчитано настолько идеально? До последней миллисекунды? Взлет, расстыковка аппаратов, выход за пределы атмосферы, вход в атмосферу Земли, ориентация на ВПП, посадка. Все это расчитано? Серьезно? Как? Это невозможно. Никто ничего подобного никогда больше ни до, ни после не сделал. Здесь столько факторов, которые нужно учесть, что от этого крышу сносит. И СССР на отстающей элементной базе смог это сделать? Как?
Если почитать заявления свидетелей Бурановых, то аппарат не просто выполнил абсолютно все по идеально заготовленной программе, он еще и подстраивался под изменения условий взлета - его программа сама совершала правильные маневры, не сделав которые аппарат бы погиб. Как? В 1988 году, да на Советских технологиях, которые не могут в серийное производство жестких дисков и микропроцессоров? Как такое возможно? Это что-то из раздела фантастики. Если это правда, то те инженеры и та страна - это действительно какой-то великий фантастический Валинор, который настолько возвышается над всем человечеством, что голову задирать устанешь, чтобы на него посмотреть.
Нет, в этом полете нет ничего "обычного". Здесь необычно абсолютно все. Поэтому я в это не верю. Всего лишь один полет? Серьезно? Никакого огромного информационного пиара на весь мир? Почему? Потому что "сука падла Горбачев", разваливая великий Союз, запретил хвастаться на весь мир, а Советские газеты были через чур заняты обсуждением рассекреченных номенклатурных дел прошлого, которые теперь стало можно обсуждать? Не верю. С такой технологией к Совку бы ломанулись все. Все захотели бы посмотреть на это чудо техники. Ведь это робот, настолько идеально реализованный на доступном ему железе, что способен сам принимать решения, как вести себя в полете, да еще и абсолютно правильные, до которых бы пилот не додумался.
Почему-то мне кажется, что это пиздешь. Слишком легко и быстро программу прикрыли. Технология "забылась", ей не воспользовалась ни Россия, ни кто-либо из стран запада (которые якобы в 90ые, при ЕБНе, массово приезжали в нашу страну, дабы скупать, скупать, скупать технические секреты всяких Яковлевых, Туполевых, Хруничевых и прочих (на этих "сданных секретах", как считают ватаны, держится теперь весь СпейсЫкс и Локхид-Мартин)). Почему? Может, ее просто не было? Вспомните, Совок довольно легко закрывал программы, в которые вложил дохуилярды ресурсов, но которые не оправдали себя. Н-1 не полетела? Ну и хуй с ним, ну и не надо. Ту-144 оказался нестабильным говном? Забьем на него большой болт спустя год с момента ввода в эксплуатацию и сгноим его в ангарах. А тут вдруг появляется невероятная технология, один единственный раз показавшая себя настолько совершенной и идеальной, что ей... просто решили никогда больше не пользоваться. Как-то слишком похоже на наебалово, не находите?
Аноним 26/03/19 Втр 21:05:07 473431267
>>473430
Не читал, просто нахуй иди.
Аноним 26/03/19 Втр 21:14:37 473434268
>>473380
Ну допустим у тебя чудо-движки держат тебя и ты не улетаешь с орбиты нахуй к альфе центавра.

>вот при такой скорости на орбиту этого корабля масса планеты сильное влияние будет оказывать?
Такое же, как и при нормальной орбитальной. Гравитация не меняется от скорости движения тел друг относительно друга, лишь от расстояния между ними. А для релятивистских эффектов, ежели таковые вообще будут, 120км/с недостаточно, это считай стоишь на месте.
Аноним 26/03/19 Втр 21:22:42 473435269
>>473430
Долбоебина этот буран потом лет 15 в ангаре стоял и на него мог придти посмотреть кто угодно
Аноним 26/03/19 Втр 21:25:34 473436270
>>473435
А кто-то сказал, что самого аппарата не было? Я такого не говорил.
Ну и как? Ходил, смотрел? Лично видел, как он без пилотов из ангара вылетает и обратно залетает?
Аноним 26/03/19 Втр 21:30:22 473440271
Аноним 26/03/19 Втр 21:30:54 473441272
>>473430
Буран — нелепая и смешная подделка под Шаттл, апофеоз советского карго-культа.

Чего стоит только провал разработки советской копии SSME, после чего пришлось отказаться даже от шаттловского частичного реюза, снять двигатели с самого челнока и перенести их на одноразовый внешний бак.

Казалось бы, понятно, что после этого в болванке Бурана больше никакого смысла не осталось, он превратился просто в ебически дорогой обтекатель массой под 70 тонн, но нет, престарелым пердунам из партии тоже хотелось иметь собственный космический самолетик.
Аноним 26/03/19 Втр 21:32:22 473442273
>>473436
просто иди сдохни в канаве
никто ничего не должен доказывать шизофренику с позицией "бля ну не верю не может такого быть мне с дивана кажется что было вот так"
Аноним 26/03/19 Втр 21:34:30 473443274
>>473441
Все верно, но какое это имеет отношение к бредням дауна из зогача?
Аноним 26/03/19 Втр 21:35:37 473444275
>>473440
>>473442
Ну понятно. Нести всякую хуету про "Допустим, у меня есть космолет, летящий на скорости 10000000000000000000 км/секунду прямо в Uranus" и засорять тренд "смешными картинками" можно, а пытаться вести дискуссию, почему вундервафля, вершина развития беспилотных технология, созданная 30 лет, летала только 1 раз и не получила больше нигде никакого развития - нельзя.
Демократия на дваче.
Аноним 26/03/19 Втр 21:37:38 473445276
>>473444
Зогачеры невменяемые животные, которые почти как религиобляди. Если они хотят верить во что-то, то с ними нет смысла говорить, они срут не для того чтобы на их вопросы ответили и помогли им что-то понять, а чтобы срать и распространять свою шизофрению и религию.
Аноним 26/03/19 Втр 21:47:12 473449277
>>473444
Потому что блядь у тебя в вопросе уже заложено утверждение. Так только конченные ублюдки делают.
Если бы нормально спросил "а правда что автопосадка бурана это невероятное техническое достижение?" тебе бы нормально пояснили что мол так и так, система сложная и является достижением, но ничего невероятного нет и вообще буран говно.
Но ты блядина даже не допускаешь что система реальна и не пытаешься сперва разобраться, а вместо этого начинаешь нести зогоблядский бред по пилотов внутри при этом явно нихуя не понимая в вопросе.
Тупые вопросы тут разрешены, а тупые долбоебы нет.
Аноним 26/03/19 Втр 22:00:25 473455278
>>473449
>тебе бы нормально пояснили что мол так и так, система сложная и является достижением, но ничего невероятного нет и вообще буран говно.
>система сложная и является достижением, но ничего невероятного нет
>ничего невероятного нет
Вы че блять, троллите что ли? Вы что блять, суки, несете? Аппарат блять, в 1988 году сука, сам берет, взлетает блять, сам хуярит 2 витка вокруг Земли, сам входит в атмосферу (да не так блять, как Союзы или Аполлоны делают, где огромная погрешность в точке входа, что они потом могут сесть в лесные/океанические ебеня, где их будут искать еще несколько часов, а очень четко, спланировано и скоординировано), сам совершает охуительные предпосадочные маневры с ориентированием на ВВС с разворотами почти под прямой угол, сам садится на ВВП, сам блять выпускает ебаный парашют. И ВЫ ГОВОРИТЕ "НИЧЕГО НЕВЕРОЯТНОГО БЛЯТЬ НЕТ". ВЫ ЧТО, ЕБАНУТЫЕ?
ЭТО ОХУЕТЬ КАК НЕВЕРОЯТНО. ЭТО ПИЗДЕЦ КАК НЕВЕРОЯТНО.
Блять, на фоне этого любая беспилотная технология прошлого, настоящего и будущего сосет с таким глубоким заглотом, что про них даже говорить не надо. Буран все сделал раньше и настолько идеально, что все. Совершенство. Предел достигнут.
Ну, и его сразу же сворачивают после 1го полета, закатывают в ангар, где он сгнивает. Охуеть блять. Ничего невероятного. Пиздец. Ничего блять. Невероятного. Я в ахуе. Ничего.
Вы блядь меня троллите.
Аноним 26/03/19 Втр 22:01:32 473456279
>>473455
>ВВС
ВВП*
Извините.
Ничего необычного блять.
Аноним 26/03/19 Втр 22:05:06 473457280
>>473456
Да блять.
ВПП*. Какой нахуй ВВП.
Извините.
Невероятного блять ничего.
Аноним 26/03/19 Втр 22:09:28 473459281
>>473455
>Аппарат блять, в 1988 году сука, сам берет, взлетает блять, сам хуярит 2 витка вокруг Земли
Как блядь это влияет на общую сложность, а?
>сам входит в атмосферу
держать угол атаки даже автопилот кукурузника может
>да не так блять, как Союзы или Аполлоны делают, где огромная погрешность в точке входа, что они потом могут сесть в лесные/океанические ебеня
А ничего бля это по сути самолет и он может планировать на тысячи км к любую сторону, в отличии от неуправляемых банок?
>сам совершает охуительные предпосадочные маневры с ориентированием на ВВС с разворотами почти под прямой угол
>сам садится на ВВП
Вот тут как раз самая сложность, но ничего невозможного.
>сам блять выпускает ебаный парашют
Не ну это пиздец сложнейшая операция что у тебя в голове вообще
>Блять, на фоне этого любая беспилотная технология прошлого, настоящего и будущего сосет с таким глубоким заглотом, что про них даже говорить не надо. Буран все сделал раньше и настолько идеально, что все. Совершенство. Предел достигнут.
Шатол мог, но пилоты бунтовали и саботировали автоматические посадки.
>Ну, и его сразу же сворачивают после 1го полета, закатывают в ангар, где он сгнивает. Охуеть блять. Ничего невероятного. Пиздец. Ничего блять. Невероятного. Я в ахуе. Ничего.
Ну да бля вот у нас дорогое беззадачное говно, давай пускать его чисто потому что он может сам садиться и похуй что у страны денег нет а нам надо как-то Мир держать, что у тебя в голове вообще х2
Аноним 26/03/19 Втр 22:17:17 473466282
>>473459
>держать угол атаки даже автопилот кукурузника может
Представил себе кукурузник, входящий в атмосферу в облаке плазмы и проиграл. Пойду в KSP строить.
Аноним 26/03/19 Втр 22:27:30 473472283
>>473455
А ты в курсе, что баллистические ракеты тоже сами по звездам ориентировались в то время и сами на цель выходили, корректировали свой полёт? В ебаной ракете и боеголовке мозги уже кое какие были.
Аноним 26/03/19 Втр 22:32:11 473473284
>>473472
Ну-ка, поподробнее. Никогда о таком не слышал, куда гуглить?
Аноним 26/03/19 Втр 22:36:11 473474285
>>473473
Поддерживаю этого огурца.
Аноним 26/03/19 Втр 22:41:00 473477286
image.png (227Кб, 594x380)
594x380
Ох-хохо. Кажется, в треде
БУРЯ
У
Р
Я

с ахуенной полностью автономной навигацией по звёздам на элементной базе 50-х.
Аноним 26/03/19 Втр 22:42:03 473478287
>>473473>>473477

Вот ты задаешь ракете чтобы она упала на координаты хуй хуй хуй такой долготы, хуй хуй хуй такой широты, а как по твоему она узнает где она находится и где её цель? GPS тогда не было, а падать на столицу врага надо. Лазером никто столицу врага не подсвечивает, на радиовышку она не наводится. Уже значит нужно как-то ориентироваться по ориентирам - на местности и по звездам.
Аноним 26/03/19 Втр 22:45:48 473479288
>>473477
>на элементной базе 50-х.

В космосе небо всегда чорное, значится нужна одна камера с узким полем зрения, которая будет пырить на звезды и распознавать среди них по светимости хотя бы три яркие на заданном расстоянии. Просто представь что ты делаешь в бумажке 3 дырки, потом ночью смотришь на небо через эти три дырки до тех пор, пока они не совпадут с 3 звездами из пояса ориона. Всё.
Аноним 26/03/19 Втр 22:49:52 473481289
>>473441
>после чего пришлось... снять двигатели с самого челнока и перенести их на одноразовый внешний бак
Глушко задумал ряд унифицированных РН тяжёлого и сверхтяжёлого класса вообще ещё до программы МТКС. Это ракетчикам удалось убедить военных сделать так, как в итоге сделали, а не «не получилось».
>провал разработки советской копии SSME, после чего пришлось...
Даже если принять, что это так, то всё равно связи нет.
Аноним 26/03/19 Втр 22:50:24 473482290
>>473477
В 50-х? Когда в СССР (да и вообще в мире) транзисторные компьютеры в зачаточном состояние? Ламповый бортовой компьютер что ли был?
Аноним 26/03/19 Втр 22:50:46 473483291
>>473479
Вот только летела она не в космосе.
А я и не говорю, что это какая-то советская магия. Это просто пример того, как лаптями и изолентой можно реализовать решение задачи, которая в наше айфонное время просто по умолчанию кажется требующей таких-то компьютерных технологий.
Аноним 26/03/19 Втр 22:52:38 473484292
А вот ещё вопрос.
Если у нас наклонённая орбита относительно плоскости экватора, спутник летает с постоянной скоростью. А вот если смотреть его скорость относительно отрезка, образуемого пересечением плоскостей экватора и орбиты. там скорость будет равномерной по всему отрезку, или она меняться будет?
Аноним 26/03/19 Втр 22:54:16 473485293
>>473482
Ну первые полупроводники уступали в характеристиках лампам. И с нагревом у них было, к тому же, хуже.
Аноним 26/03/19 Втр 22:57:26 473486294
>>473483
Ебать. Какая-то через чур вундервафля для 50х годов. Где взять документацию с подробным описанием, как работала система ориентирования по звёздам?
Аноним 26/03/19 Втр 22:59:50 473487295
>>473484
Постоянной не будет. Если скорость движения по орбите постоянная, значит орбита круговая, то будет синусоидально меняться. В общем случае закон изменения скорости проекции на эту линию пересечения плоскостей зависит от значений эксцентриситета и аргумента перицентра орбиты.
Аноним 26/03/19 Втр 23:02:35 473488296
>>473486
>Где взять документацию с подробным описанием
Копай, если тебе интересно и если не засекречено, то можно найти. Лозга постоянно писал про это в блоге по тегу "незаметные сложности" можешь там в прошлое пролистывать и рано или поздно наткнёшся на нужный материал
https://lozga.livejournal.com/tag/незаметные сложности
Аноним 26/03/19 Втр 23:04:57 473490297
>>473486
Практически все источники, которые я видел (а я, справедливости ради, не очень-то тему и чекал), ссылаются вот на это: http://epizodyspace.ru/bibl/evstafiev/evstafiev_0.htm
Там ищешь описание работ собственно над Бурей и Бураном нет, не тем, там и про навигационную систему есть.
Аноним 26/03/19 Втр 23:09:33 473491298
>>473486
Да не вундервафля это, забей. Спиздили идею у Northrop'а, только реализовать не смогли и эти ракеты на вооружение не приняли, в отличие от Нортроповских.
Аноним 26/03/19 Втр 23:18:45 473493299
>>473455
Чего ты так бомбишь, есть и сейчас подобные технологии, даже круче, которые реально эксплуатируются.
https://ru.m.wikipedia.org/wiki/Boeing_X-37
Не создали их американцы раньше, потому что решили по концептуальным причинам не делать Шаттл полностью беспилотным. Им тогда это было нинужно. Как стало нужно - сделали.
Совок же заморочился и умер от экономического разрыва жопы.
Аноним 26/03/19 Втр 23:26:59 473495300
>>473491
>Спиздили идею у Northrop'а
Буря куда больше походит на Навахо.
>реализовать не смогли
Хуя не смогли - летала же.
>эти ракеты на вооружение не приняли
Потому что не нужны при наличии МБР. Американцы это тоже поняли и свернули все свои аналогичные работы. Снарк был не нужен точно так же, но ему как-то повезло.
Аноним 26/03/19 Втр 23:29:39 473496301
RetirementAtlan[...].jpg (211Кб, 1000x667)
1000x667
>>473455
Ну ёба, а Шаттл чем занимался ещё с 81-го? Или ты думаешь, что там обезьяны вручную маневрировали на взлёте и входе в атмосферу, самых сложных и опасных участках полёта, где малейшая ошибка приведёт к молниеносному нарезанию твоего ануса на мелкую лапшу?
Тот же Шаттл обычно разве что на глиссаду автоматически не заходил, однако даже на этапе CSS (переключение на ручное управление, примерно за 3 минуты до посадки) компьютер выполняет все операции по расчёту траектории и показывает, куда и как надо лететь: фактически всё, что нужно делать капитану, это держать нос Шаттла в высвеченном на HUD квадратике.
Кстати, собственно автолэнд, работающий до момента касания полосы, в Шаттлах тоже был (не окончательно доведённый до ума из-за баттхёрта пилотов, но полностью рабочий - о нём говорится на 25 странице Space Shuttle Avionics System и в разделе 7.4-25 Shuttle Crew Operations Manual) - например в полёте STS-3 проводились его испытания, и пилоты перешли на ручное всего за 40 метров от земли после чего сами обосрались и чуть не разъебали переднюю стойку шасси
Аноним 26/03/19 Втр 23:42:07 473499302
>>473455
Безграмотное хуйло. Самолёты могли садиться автоматически задолго до Шаттла, вообще-то.

У Шатола человек нажимал на ряд кнопочек, вроде выпуска ПВД, из-за давления астронавтов. Да, так бывает, даже про традицию с певых дней космической программы говорить можно. Сейчас такой хуйни нет, наконец-то.
Более того, код для управления этими функциями был написан.
После "Колумбии" добавили кабель от стоек с компьютерами к этим кнопкам, чтобы автоматически замыкать.

Ну и самый главный вопрос: ты действительно думаешь, что человеку доверили бы управлять сходом с орбиты? Даже если стабилизировать на горячем участке автоматически - лысая обезьяна просто не сможет вывести эту хуйню на глиссаду. Чуть не там повернешь на такой высоте и скорости - перелетишь/недолетишь до полосы, банально, без второй попытки.

Сука, как же заебали, безграмотные пидорасы не могущие в Гугл. Великое достежение у него нахуй, телевизор смотреть меньше надо, блядь.
Аноним 26/03/19 Втр 23:47:01 473500303
>>473483
>лаптями и изолентой можно реализовать решение задачи, которая в наше айфонное время просто по умолчанию кажется требующей таких-то компьютерных технологий
Только вот точность будет оче хуёвой.
Прошивку не проапдейтишь просто так.
Перспектива дебага даже на стендах вызывает мысли о суициде.
А еще всё это в габаритах твоей мамки хуй знает чего.

Нутыпонел, да?
Аноним 26/03/19 Втр 23:50:31 473501304
>>473495
SLAM гугли. Вот це да, пиздец.
Аноним 27/03/19 Срд 00:56:19 473507305
15192908923280.jpg (14Кб, 536x291)
536x291
>сага о том, как один зогачер затраллировал пейсач
Ебать вы дебилы, нахуй вы кормите этот биомусор, я не понимаю. Ладно бы он тупой вопрос задал, и то пиздец, но он же пришёл сюда нести свет.
Аноним 27/03/19 Срд 04:06:23 473521306
>>473481
Связь очень простая: изначальный проект корабля ОС-120 как раз полностью копировал компоновку шаттла и имел три двигателя в жопке челнока, но потом оказалось, что советский челнок выходит тяжелее, массовое совершенство баков хуже, двигатели имеют меньшую тягу (на уровне моря SSME — 1860 кН, советский аналог РД-0120 — 1526 кН, почти на 20% меньше) при той же массе, и на трех двигателях Буран просто неспособен выйти на орбиту.

Для четвертого двигателя в челноке не было места, да и сам корабль становился уже совсем неприлично тяжелым, поэтому ОС-120 был списан в утиль и был принят новый проект, с двигателями на баке второй ступени, который и был построен. Сделать многоразовость как на Шаттле советские конструкторы тупо не могли, даже если и хотели.
Аноним 27/03/19 Срд 05:17:50 473522307
>>473496
>после чего сами обосрались и чуть не разъебали переднюю стойку шасси
Ну как сказать обосрались, просто багопилот из-за осцилляций вынудил садиться на 500км/ч. Некоторые самолёты даже летать так быстро не умеют, а самые быстрые посадочные скорости для самолетов это 300-320км/ч. У Шаттла стандартные скорости в районе 350-370, редко выше.

Лишь один ЛА когда-либо отдаленно приближался к такой посадке уровня KSP - бешеный гроб F-104 с крыльями как у гвоздя садился на 440км/ч, и то лишь при сломанном воздушном тормозе, который впрочем постоянно ломался.
Аноним 27/03/19 Срд 06:39:26 473525308
>>473521
Давай источник. У Губанова написано, что четвёртый двигатель блока Ц - практически горячий резерв.
Аноним 27/03/19 Срд 10:06:23 473543309
27/03/19 Срд 10:14:06 473544310
>>473543
а самому типа сложно прочитать и понять? Ты частица?
Аноним 27/03/19 Срд 10:15:54 473545311
>>473496
Нихуя. Это выходит, что Шаттл на охуенной скорости при охуенной скорости ветра сам зашел на посадку на не типичную для него полосу на соленом озере (понятное дело, что туда всю аппаратуру привезли, но Шаттлы не каждый день садятся на соленое озеро) и передал управление хьюманам только уже на осуществление непосредственно самого касания посадочной полосы?
Ебать какие ебатехнологии-то блять. Это круто. Не, понятно, что из-за самовольности софта Шаттл через чур развил большую скорость и чуть не разъебал пассажиров-мартыханов. Но все ведь закончилось хорошо.
Аноним 27/03/19 Срд 10:29:03 473546312
Аноним 27/03/19 Срд 11:05:15 473550313
Расскажите подробнее за компьютер ПС-2000, который обрабатывал фотки с аппаратов Венера-15/16. Я так понял, что это был аппарат получше, чем говноЭльбрус. Во всяких статьях пишут, что он достигал то 200 MIPS, то 1000 MIPS в середине 80х. Где правда?
Он был собственной разработки или скопипизженный?
Аноним 27/03/19 Срд 11:37:08 473556314
>>473550
Да забей хуй. Говно все эти совсковские чудо-компьютеры. Все в них от и до спизженно. Архитектуры они, конечно, могли какие-нибудь свои придумывать (так и было, их было дохуя, большинство из них такое говно, что история стыдливо затерла упоминания о них, а те, что были успешными - ну, они спизженные, конечно), но элементная база-то все равно была полностью спизжена. Конкретные модели транзисторов, процессоры, железо - блоки памяти, жесткие диски, терминалы ввода/вывода, - софт - в первую очередь архитектуры языков программирования и операционные системы, - все это коммуниздилось только в путь. Некоторые ебалаи вроде какого-нибудь Бабаяна еще имеют совесть при этом утверждать, что это они "все разработали". Дебилы, блять.
Аноним 27/03/19 Срд 11:45:24 473557315
>>473545
Какой-то ты впечатлительный.
Аноним 27/03/19 Срд 19:53:32 473618316
Раз за компы совковские пошел пиздешь - вычитал в википедии, что совковский суперкомпьютер М-13 (это, кстати, не совсем оффтоп: этот аппарат был произведен в кол-ве 20 штук и применялся в советских ПВО - за ракеты ПВО мы же тут тоже пиздим, верно?) с 1985 по 1990 год мощнейшим в мире. Даже мощнее, чем Cray-2 и Cray Y-MP. Полнейший пиздешь и провокации или не совсем?
Аноним 27/03/19 Срд 19:55:40 473619317
>>473618
А параметры сравнить слишком сложно? Или у совкового компа они засекречены?
Аноним 27/03/19 Срд 20:04:55 473620318
>>473619
Бля, да там очень лулзово по параметрам написано. В одном месте написано 1 GFLOPS, в другом 2 GFLOPS, а в ТРЕТЬЕМ БЛЯТЬ 2,4 GFLOPS. Охуенный распрыг. Не удивлюсь, если на самом деле 0 с хуем GFLOPS.
А официально, наверное, засекречено. По крайней мере я не нашел.
Вообще, там спиздануто, будто бы это первый компьютер, перешедший рубеж в 2 GFLOPS'а (хотя в другом месте написано, как я указал выше, что он был 1 GFLOPS... сука блять). Может, от этого погуглить.
Аноним 27/03/19 Срд 20:10:57 473622319
>>473556
Так тут главный лулз в чем - спизжено-то спизжено, но спиздить это - это был охуенный труд. Это же нужно взять какой-нибудь микропроцессор, устроить его полную разборку, понять как работает, почему работает, как сделан. И попытаться повторить.
Это был охуительно сложный реверс-инжениренг. Ведь кроме вышеперечисленного, чтобы создать хотя бы опытный образец, нужно еще и создать оборудование, которое его воспроизведет в таком виде, как у западного оригинала. Это пиздецки сложно. Они по сути каждый западный оригинальный элемент заново переизобретали.
И совок даже в целом более-менее успешно справлялся, никогда не отставал больше, чем лет на 5.
Но когда в 80ые дело пошло в массовое промышленное производство на народные нужды, где объемы должны были быть миллионными - это стало еще одной причиной смерти совка от экономического разрыва жопы.
Аноним 27/03/19 Срд 20:11:40 473623320
>>473620
наверняка 1 - рабочая цифра, 2 - предельные параметры на которых устройство живет минуту, 2.4 - взрывается к хуям
Аноним 27/03/19 Срд 22:55:26 473658321
Возможно ли существование целого кольца на орбите планеты? Не кольца из камней, как у Сатурна, а целого монолитного вращающегося кольца.
Примерно чувствую, что такое кольцо существовать не сможет, но не могу объяснить почему:

В принципе, целое монолитное кольцо можно представить состоящим из отдельных близких кусков, летящих по орбите, и тогда кольцо должно быть стабильным.

Еще можно представить, что мы берем кольцо как у сатурна, и начинаем протягивать между отдельными глыбами тонкие нити. На этом этапе ничего не должно произойти. Начинаем постепенно утолщать нити, пока помностью не заполним промежутки между камнями. В какой момент все это обрушится?
Аноним 28/03/19 Чтв 01:56:31 473667322
>>473658
В первую же секунду. У тебя ошибка в самом начале, камни на разной высоте движутся вокруг планеты с разной угловой скоростью, они не неподвижны относительно друг друга.

Можно хотя бы на орбитальные периоды спутников Земли посмотреть: 90 минут на НОО, 24 часа на ГСО, 27 суток на орбите Луны, понятно, что никакого стабильного положения относительно друг друга тут быть не может.
Аноним 28/03/19 Чтв 01:59:51 473668323
Если б у солнца был близнец, на каком минимальном расстоянии они могли б вращаться, чтоб продержаться до конца главной последовательности без существенного перетекания вещества с одной на другую звезду?
Аноним 28/03/19 Чтв 07:41:53 473680324
>>472845
Ну, Юпитер в два с половиной раза массивнее, чем все планеты нашей солнечной системы вместе взятые, если ты хочешь с ним проводить какие-то манипуляции при помощи ядерной бомбы, то эта ядерная бомба должна быть хотя бы размером с какой-нить спутник или маленькую планету.
Аноним 28/03/19 Чтв 07:54:25 473681325
>>473667
А если кольцо будет тонкое, как провод, и расположено на строго круговой орбите?
Аноним 28/03/19 Чтв 13:18:48 473709326
>>473681
Нестабильно. Быстро врежется в Землю, если его как-то активно не удерживать. Подумай, что будет, если его слегка подтолкнуть и геометрический центр кольца не будет совпадать с центром масс Земли.
Аноним 28/03/19 Чтв 13:31:36 473712327
>>473709
Можно провести его перпендикулярно экватору и центрировать двигателями с магнитными датчиками в районе геомагнитных полюсов.
Аноним 28/03/19 Чтв 17:13:32 473741328
>>473668
Да хуй знает. Вроде как Солнце стабильное и не пульсирует, так что должно быть все время одинакового размера с близнецом. То есть, если они не соприкасаются, то и перетекать у вещества причины нет. Так что я б взял утроенный максимальный радиус до начала стадии красного гиганта - хз, насколько он там больше текущего радиуса. (Утроенный для исключения потери орбитальной скорости из-за трения об атмосферу близнеца с последующим падением на него.)
Аноним 28/03/19 Чтв 17:16:55 473742329
>>473741
А, ну и еще при таком близком расположении они уже не будут шарами, а с нехилым "выростом" в направлении друг друга. На это тоже нужен какой-то запас в расстоянии, чтобы банально не ебнулись за миллиарды лет.
Аноним 28/03/19 Чтв 18:01:17 473748330
Аноним 28/03/19 Чтв 18:49:25 473759331
Кто-нибудь может объяснить, как Шаттл выводил полезную нагрузку на орбиту? У него же груз лежит в кузове, который потом открывается и... что? Астронавты его руками оттуда выпихивают? Как оно работает? Как грузу придают ускорение?
Аноним 28/03/19 Чтв 18:54:34 473762332
image.png (132Кб, 300x264)
300x264
Аноним 28/03/19 Чтв 18:54:42 473763333
>>473759
Либо шаттл отлетает вниз, либо достаёт груз манипулятором.
Аноним 28/03/19 Чтв 19:12:36 473768334
>>473762
>>473763
Странная хуйня. Разве так можно точно направить объект куда надо, да еще и придать ему нужное ускорение?
Эксперты? Эксперты?
Аноним 28/03/19 Чтв 19:19:21 473769335
>>473768
Ты дебил? Этой хуйнёй грузовики к МКС пристыковывают. Или что по-твоему, мы тут пиздим тебе, на самом деле это всё заговор, шаттл не мог ничего выводить?
Аноним 28/03/19 Чтв 19:26:32 473772336
>>473768
В смысле куда надо блядь, шатол прилетает куда надо, вытаскивает рукой груз и улетает обратно, что непонятного?
Аноним 28/03/19 Чтв 19:27:45 473773337
>>473769
Э, чего истрешь блять. Ничего я такого не говорю, просто пытаюсь понять, как оно работает. С ракетой-то все понято, там ступени, вся хуйня, от последней из которых потом и отсоединяется КА и уебывает крутиться на орбите/в дальний космос. А тут как-то не понятно, как оно работает.
Аноним 28/03/19 Чтв 19:37:32 473777338
Аноним 28/03/19 Чтв 19:37:44 473778339
>>473773
Шаттл - это последняя (и первая) ступень ракеты, он блядь сам на орбите. Так понятно?
Аноним 28/03/19 Чтв 19:38:24 473779340
>>473768
>нужное ускорение
Ускорение придавать (почти) не нужно. Шатол уже летит по той орбите, на которую выводит хуйню. Нужен совсем небольшой толчок, чтобы они разошлись.
Аноним 28/03/19 Чтв 19:42:02 473781341
>>473778
Охуеть, шатол, оказывается SSTO.
STS трёхступенчатый: твердотопливные блоки, мейн энджины со сбрасывамым баком, система орбитального маневрирования.
Аноним 28/03/19 Чтв 20:20:02 473793342
>>473781
Это неправильная классификация. В системах с зажиганием на старте нескольких "ступеней" правильно говорить боковые ускорители и центральный блок.
И еще, перицентр в момент отключения ссме было 110км, если не ошибаюсь, так что омс не ступень.
Аноним 28/03/19 Чтв 20:51:16 473799343
>>473620
Только крузисом в софтваре рендеринге можно тестить, гигафлопсов кто во что горазд написать можно, вон у видях щас по несколько терафлопс, у консолек в 2000е тоже от балды писали
Аноним 28/03/19 Чтв 21:29:36 473816344
>>473793
>Это неправильная классификация
Ну поучи ещё тех, кто вообще придумал космонавтику.
>В системах с зажиганием на старте нескольких "ступеней" правильно говорить...
Определение чёткое и однозначное. Время работы различное, разные моменты отделения - значит, разные ступени.
>перицентр в момент отключения ссме было 110км, если не ошибаюсь, так что омс не ступень
После отключения RS-25 Спейс Шаттл на суборбитальной траектории. Один или два включения OMS использовались для формирования опорной орбиты.
Аноним 28/03/19 Чтв 21:50:59 473823345
>>473816
>разные моменты отделения - значит, разные ступени.
Када там RS-25 от орбитера отстреливаются?
Аноним 28/03/19 Чтв 22:07:21 473830346
На орбите Земли висит стальной стометровый шар. Разумеется, одна из его точек находится ближе к барицентру на сто метров, чем противоположная. А значит, они находятся на разных орбитак, а значит должны иметь разную орбитальную скорость. Что будет с таким шаром? Появится некий вращающий момент?
Аноним 28/03/19 Чтв 22:11:53 473832347
>>473830
>Появится некий вращающий момент?
Почему он должен появится? Тем более у шара центр масс точно в центре, вот если взять стометровую гантелю.
Аноним 28/03/19 Чтв 22:17:57 473835348
>>473816
>Ну поучи ещё тех, кто вообще придумал космонавтику
Из-за таких долбоебов как ты до сих пор мощность машин в лошадях измеряем
Аноним 28/03/19 Чтв 22:55:58 473841349
>>473830
На орбите Земли висит каменный 3460-километровый шар. Разумеется, одна из его точек находится ближе к барицентру на 1730 километров, чем противоположная. А значит, шар растягивается в сфероид, и тогда уже приобретает вращающий момент, заставляющий его всегда поворачиваться к Земле одной стороной.
Аноним 29/03/19 Птн 00:14:09 473847350
>>473830
>Что будет с таким шаром?
В шаре возникнут крошечные приливные силы всвязи с тем, что одна его часть чуть ближе к Земле и ее тянет чуть сильнее, чем дальнюю. Для ста метров стали этим можно смело пренебречь - даже на форму не повлияет заметным образом.
>Появится некий вращающий момент?
Нет силы, которая заставит его вращаться, если он остается шаром. Движение по орбите - это следствие гравитационного воздействия, а не причина, которая может вызывать что-то. Вот если приливных сил достаточно, чтобы нарушить шарообразную форму - тогда да, за тыщи/миллионы/миллиарды лет собственное вращение будет синхронизировано с периодом обращения по орбите.
Аноним 29/03/19 Птн 01:17:05 473849351
Что будет с Союзом, если он сядет на воду? Предусмотрено ли у него что-то на такой случай, или он утонет?
Аноним 29/03/19 Птн 01:36:31 473850352
>>473849
Утонуть он никак не может пока люки не откроют. Но проблема с устойчивостью может быть. Видел на какой-то фотографии внутри мешок с надписью "гидрокостюмы", но хуй знает как они их надевать будут в такой тесноте.
Аноним 29/03/19 Птн 02:14:47 473852353
>>473849
Открытый парашютный контейнер заполняется надувным баллоном с газом - "вытеснительная ёмкость" - чтобы не набрал воды.
>>473850
Не будут, а и надевают на тренировках.
Аноним 29/03/19 Птн 02:24:00 473854354
>>473849
Союз-23 садился в воду и не утонул. Правда, капсула накренилась и выходной люк под водой оказался, так что самостоятельно вылезть космонавты не могли, но утонуть не утонули.
Аноним 29/03/19 Птн 09:46:42 473880355
>>473847
>Вот если приливных сил достаточно, чтобы нарушить шарообразную форму - тогда да, за тыщи/миллионы/миллиарды лет собственное вращение будет синхронизировано с периодом обращения по орбите.
То есть спутники всех планет всегда смотрят на них одной стороной?
Аноним 29/03/19 Птн 13:07:36 473890356
>>473880
Нет, но стремятся к этому. Если дать системе планета - спутник достаточно времени (в некоторых случаях "достаточно" - это больше возраста Вселенной), то спутник будет повернут к планете одной стороной. Время "устаканивания" тем меньше, чем ближе спутник к планете, и чем массивнее планета. Если спутник неправильной (нешарообразной) формы - процесс идет еще быстрее.

Иногда, впрочем, возможны другие резонансы. Например, Меркурий: у него период собственного вращения равен 2/3 от солнечного года. Такая хуйня может возникнуть, если орбита достаточно сильно вытянута.
Аноним 29/03/19 Птн 16:41:50 473918357
Посоветуйте книгу Стивена Хокинга про пространство и время.
Аноним 29/03/19 Птн 17:00:18 473929358
Аноним 29/03/19 Птн 18:28:34 473933359
>>473918
Книга Стивена Хокинга "Краткая история пространства и времени".
Аноним 29/03/19 Птн 19:57:04 473943360
15519060719763.jpg (309Кб, 1200x889)
1200x889
При каких условиях выполнимо, чтобы вокруг звезды тороидом вращалось кольцо в сечении на несколько тысяч километров, состоящее из воздуха, плотного до уровня моря по земным меркам, и тёплого?
Какие должны быть условия, чтобы при этом воздух не сдувало прочь и не сжиживало в кучки газа?
При этом в этом же тороиде вращаются планета(ы) и целая куча астероидов, которые одновременно перемешивают газ внутри тороида, и при этом своими гравитационными влияниями не дают ему разбежаться прочь. Планета, допустим, типа Марса, но можно поболее сделать, увеличив до Земли.
Аноним 29/03/19 Птн 20:01:00 473944361
Аноним 29/03/19 Птн 20:14:05 473948362
Аноним 29/03/19 Птн 21:34:28 473977363
thinking.png (58Кб, 500x500)
500x500
Почему не изучат биологическое создание тока как это делают скаты или угри, и не сделают революцию в энергетике?
Аноним 29/03/19 Птн 21:35:37 473979364
>>473977
Потому что кпд говно
Аноним 29/03/19 Птн 21:38:25 473980365
>>473979
Я читал что угри ток такой генерируют, что крокодила йобнуть может. Значит ток там большой. Уже пытались?
Аноним 29/03/19 Птн 21:39:48 473981366
>>473980
Причем тут сила тока если я говорю про кпд? Ты что, тупой?
Аноним 29/03/19 Птн 21:42:39 473982367
Аноним 29/03/19 Птн 22:14:03 473983368
Защитит ли фул доспех обыкновенного средневекового рыцаря от спела молнии, перенаправив её в Землю?
Аноним 29/03/19 Птн 22:33:00 473986369
>>473983
Гугли "клетка Фарадея" и "Заземление"
Аноним 29/03/19 Птн 22:35:28 473987370
>>473841
Давай теперь проведем через центры Земли и Луны диаметр. На этом диаметре отметим две точки, где он пересекает луну. А теперь выкинем на хуй луну и в эти точки поместим два теннисных мяча. Они же будут двигаться с разными линейными и угловыми скоростями?
В зависимости от ответа, я задам второй вопрос.
Аноним 29/03/19 Птн 22:39:26 473988371
>>473977
Изучено, иногда используется, говно.
Аноним 30/03/19 Суб 02:41:53 474003372
>>473943
Ни при каких, в любой более-менее плотной среде планеты быстро затормозятся и упадут на звезду.
Аноним 30/03/19 Суб 09:05:36 474011373
>>473988
А где почитать про это?
Аноним 30/03/19 Суб 10:43:27 474027374
>>474011
Только вчера гулял по своему универу и видел дверь с надписью Биоэлектроника, внутри что-то булькало и воняло. Погугли это слово.
Аноним 30/03/19 Суб 11:31:52 474039375
>>474027
>внутри что-то булькало и воняло
У меня то же самое было в туалете, когда там батя сидел. Он там биоэлектроникой занимался что ли?
Аноним 30/03/19 Суб 12:05:42 474051376
>>474039
Мог бы под дверью посмотреть в щель, спросил бы что он там делает.
Аноним 30/03/19 Суб 12:35:29 474064377
>>474003
А если мы предположим, что газ не статичен и движется с такой же скоростью, что предполагаемая планета и - более того - эта планета как бы и есть центр этого кольца газа, то есть распыление таких объёмов газа началось с выпаривания газа с планеты. Тогда планета не сможет тормозить о свою собственную атмосферу также, как Земля не замедляет своё вращения из-за трения о земную же атмосферу.
Аноним 30/03/19 Суб 13:20:28 474075378
>>473987
С разными. А еще, оба будут двигаться не по круговым орбитам.
Аноним 30/03/19 Суб 17:50:42 474199379
Где найти дохуя миллионов фоток с программы Аполлон в шикарном HD качестве?
Аноним 30/03/19 Суб 17:58:54 474201380
Аноним 31/03/19 Вск 11:15:17 474301381
>>473943
>кольцо в сечении на несколько тысяч километров, состоящее из воздуха, плотного до уровня моря по земным меркам, и тёплого?
Это просто невероятные объемы и массы газа, которых хватит на тысячи планет размером с Юпитер. С одной планеты не выпарить.
>при каких условиях
Если какие нибудь могущественные инопланетяне будут искусственно поддерживать эту структуру.
Аноним 31/03/19 Вск 21:56:34 474402382
Почему не развивают ядерные ракетные двигатели? Ведь все понимают, что из химических двигателей большего, чем сейчас, удельного импульса не выжать как ни старайся. Единственный выход для человечества - ядерная и термоядерная энергия.
Аноним 31/03/19 Вск 23:41:44 474411383
>>474402
Из-за радиоактивного загрязнения атмосферы.
Аноним 01/04/19 Пнд 03:14:03 474421384
>>474075
Почему не по круговым? Гравитация между мячами влияет?

Но мой вопрос - почему тогда Луна не испытывает вращающий момент, связанный с тем, что «дальний» полюс пытается сьебаться от ближнего. Этот момент должен быть в плоскости эклиптики, но его нет.
Аноним 01/04/19 Пнд 08:52:41 474431385
>>474411
Чушь, никакого загрязнения нет при закрытом теплообменнике. Да и испытания можно в космосе производить, а не в атмосфере.
Аноним 01/04/19 Пнд 09:38:04 474438386
>>474402
Развивают, только там есть миллион проблем, которые не дают построить такой дрыгатель прямо завтра. Самое первое что приходит на ум- отвод тепла.
Аноним 01/04/19 Пнд 09:50:54 474442387
>>473943
При условии наличия плотных герметичных стенок у такого тора. Ну или при наличии в середине этого газового облака бублика из материи, создающей притяжение на уровне земного. Иначе как это облако удержать при такой плоскости? Оно мгновенно начнёт рассеиваться до средней плотности межпланетного вещества.
Аноним 01/04/19 Пнд 10:24:37 474448388
Аноним 01/04/19 Пнд 10:31:27 474450389
Аноним 01/04/19 Пнд 10:58:25 474452390
>>474448
Энергетическая установка мегаваттного класса
Аноним 01/04/19 Пнд 11:02:22 474453391
>>474438
Когда делали аполлоны и летали на луну, тоже дохуя проблем было, но смогли же.
Аноним 01/04/19 Пнд 11:39:03 474457392
>>474402
Потому же, почему закрывают ядерные станции. Зеленобляди-жопоголики.
Аноним 01/04/19 Пнд 11:53:00 474460393
>>474453
Ну если также напрячься, то лет за 10 могут и родить.
Аноним 01/04/19 Пнд 12:32:24 474467394
Worldsanimate.gif (365Кб, 480x480)
480x480
А если двое вылетают с Земли в противоположном направлении со скоростью с/2. Относительно друг друга у них скорость света. Что тогда будет со всеми этими релятивистскими эффектами? Не перемудрил ли Эйнштейн?
Аноним 01/04/19 Пнд 12:46:39 474468395
>>474467
Откуда вы лезете то, блядь?
Аноним 01/04/19 Пнд 12:48:44 474470396
>>474467
Да, а если будут лететь со скоростью 0.51с, то относительно друг друга будет 1.02с, беги скорее публиковать, теория относительности опровергнута.
Аноним 01/04/19 Пнд 12:58:32 474472397
>>474421
>Почему не по круговым? Гравитация между мячами влияет?
Ближний будет двигаться со скоростью, недостаточной для круговой орбиты на этом расстоянии - значит будет приближаться к Земле. Дальний - наоборот. Короче, оба будут на эллиптических орбитах, причем в точке, где ты их "отпустишь" между орбитами будет минимальное расстояние, равное диаметру Луны.

>Но мой вопрос - почему тогда Луна не испытывает вращающий момент, связанный с тем, что «дальний» полюс пытается сьебаться от ближнего.
Потому что
>Движение по орбите - это следствие гравитационного воздействия, а не причина, которая может вызывать что-то.
Есть такая хуйня как момент импульса. Точно так же, как тело без влияния внешних сил движется равномерно и прямолинейно, тело без влияния сил, приложенных не к центру масс, вращается с постоянной скоростью (может и нулевой) и осью. В случае, если на орбите строго шарообразное тело с радиально-симметричной плотностью, гравитация идет строго через центр масс и ничего не вращает. Другое дело, что в реальности малые тела обычно далеки от шара, а большие испытывают деформацию от приливных сил, поэтому вращающее действие возникает. В том числе и на Луну, кстати.
Аноним 01/04/19 Пнд 13:14:22 474476398
>>474470
А если ракета движется со световой скоростью и космонавт захочет переместится из хвоста на нос, его аннигилирует?
Аноним 01/04/19 Пнд 14:00:11 474491399
>>474476
Ракета не сможет двигаться со световой скоростью.
Аноним 01/04/19 Пнд 14:08:13 474492400
Мимо Земли пролетает ракета со скоростью почти с, в ней сидит космонавт и в определенный момент светит фонариком на другую планету, расположенную в 1 световом годе от Земли и ракеты. В какой момент времени пятно света на этой планете увидят жители этой планеты, земляне и космонавт?
Аноним 01/04/19 Пнд 14:12:10 474493401
>>474492
Свет всегда распространяется только со скоростью света.
Аноним 01/04/19 Пнд 14:19:15 474494402
>>474493
Относительно чего?
Аноним 01/04/19 Пнд 14:21:09 474495403
>>474494
Относительно наблюдателя.
Аноним 01/04/19 Пнд 14:35:17 474497404
Аноним 01/04/19 Пнд 14:40:03 474498405
Аноним 01/04/19 Пнд 14:56:05 474502406
Аноним 01/04/19 Пнд 14:56:58 474503407
Аноним 01/04/19 Пнд 15:03:13 474504408
>>474492
>жители этой планеты
Через 1 земной год, если планета не имеет огромной скорости относительно Земли.
>земляне
Через 2 (при аналогичных условиях).
>и космонавт?
Зависит от того, насколько его скорость близка к c. Хоть через час. Гугли релятивистское сокращение расстояния.
Аноним 01/04/19 Пнд 15:21:05 474513409
>>474504
А если светить в строну противоположенную движению?
Аноним 01/04/19 Пнд 16:55:47 474531410
>>474513
Бля, тебе уже сказали, свет летит с одинаковой скоростью относительно любой системы отсчёта, его скорость не меняется при переходе к другой системе отсчёта, он пролетает один световой год за один год, всегда.
Аноним 01/04/19 Пнд 19:27:03 474556411
ГОРЯЧЕЕ И ХОЛОДНОЕ РАЗДЕЛЕНИЕ СТУПЕНЕЙ
Как думаете что целесообразнее использовать и за чем будущее
Лично я за горячее, так как решается проблема осадки, но нужна теплозащита что вполне решаемо
Какие есть мысли и идеи по поводу?
Аноним 01/04/19 Пнд 19:33:41 474562412
>>474556
Конечно, холодное. Если на верхней ступени все равно будут стоять двигатели малой тяги, которых хватит для осадки, зачем лишний раз коптить нижнюю? Ее реюзать еще, как-никак.
Аноним 01/04/19 Пнд 19:46:01 474572413
>>474199
Рутрахер
У
Т
Р
А
Х
Е
Р

50 гигов фотачек
Аноним 01/04/19 Пнд 19:54:41 474575414
>>474556
горячее для долбоебов с технологиями 60х годов у которых ракета из 3 ступеней не способна в орбиту дальше ноо, поэтому надо сверху ставить еще 4 ступень разгонный блок.
Аноним 01/04/19 Пнд 21:45:38 474608415
Что такое снеговая линия?

Пожалуйста, объясните вселенской тупице
Аноним 01/04/19 Пнд 21:50:21 474610416
>>474608
Расстояние от звезды где она настолько слабо греет, что вода замерзает
Аноним 02/04/19 Втр 00:16:19 474617417
>>474472
Вот теперь понял, благодарю. Гоняю теннисные мячи в ксп, пухнет голова.
Аноним 02/04/19 Втр 08:40:51 474641418
>>474610
Замерзание воды зависит от давления атмосферы. Или ты имеешь в виду место, где вода уже не может существовать в жидкой фазе?
Аноним 02/04/19 Втр 11:12:50 474653419
Какой мощности заряд нужен, чтобы получить полынь во льдах Европы? И сколько она просуществует?
Аноним 02/04/19 Втр 11:47:06 474660420
>>474653
Там и без этого трещины образуются из-за гравитационного влияния Юпитера.
Аноним 02/04/19 Втр 11:47:14 474661421
>>474653
зависит от толщины льда. Мощность заряда в таких делах не важна, важна масса болванки и скорость при столкновении.
Толщина льда неизвестна точно, может она 10 км, может в каких-то местах 3 км? Пока средняя толщина, географические координаты тонких мест, вязкость льда, его слоистость - пока всё это неизвестно точно ответить на вопрос нельзя.
Аноним 02/04/19 Втр 15:22:30 474703422
>>474653
>Какой мощности заряд нужен, чтобы получить полынь во льдах Европы? И сколько она просуществует?
Чтобы получить полынь в Европе, кроме льда нужен грунт, теплый воздух, освещение и семена полыни. Соотвествтвенно заряд нужен такой мощности, чтоб освещать и обогревать микропарничок с растением в течении всего его срока жизни.
Полынь вроде однолетняя, так что вряд ли дольше одного года просуществует.
Аноним 02/04/19 Втр 16:05:30 474714423
>>474703
Нет, для этого нужно, чтобы Агнец снял седьмую печать, и вострубил сначала первый ангел, затем второй, и потом третий. Тогда звезда Полынь падёт на источники вод, и третья часть вод сделается полынью. Объём океана жидкой воды на Европе оценивается в 3 * 10^18 кубометров, соответственно заряд должен быть достаточно массивным, чтобы радиоактивных продуктов от взрыва хватило, чтобы отравить 10^18 кубометров воды.
Аноним 02/04/19 Втр 16:16:42 474716424
>>474641
Снеговая линия находится гораздо дальше, чем линия тройной точки воды (которая вообще внутри орбиты Земли).

Имеется в виду прямой процесс сублимации-десублимации, т.е. такая температура, при которой водяной пар выпадает в лед и не возгоняется обратно даже под воздействием солнечных лучей. На таком расстоянии лед может слипаться в снежинки, и даже собираться в ледяные астероиды и более крупные тела, а ближе к Солнцу они даже при <0°C начинают пердеть как кометы.
02/04/19 Втр 17:27:39 474724425
>>474716
ну сублимация из-за солнечного излучения в том или ином рейте происходит далеко за снеговой линией. Просто из-за отдельных высокоэнергетичных солнечных частиц и медленно, очень медленно, хотя со временем space weathering все равно накапливается эффект
Аноним 02/04/19 Втр 18:22:09 474735426
>>474661
ну так примерно почувствовать можешь
Аноним 02/04/19 Втр 18:58:34 474736427
1466360608497.jpg (396Кб, 600x600)
600x600
>>474735
>примерно почувствовать можешь
Не могу, слишком далеко Европа. Погугли такой кратер Пвыл(или Пуйл) на поверхности Европы. Астероид его оставивший может пробил до океана полынью.
В любом случае, пробивать поверхность не обязательно. В ближайшие 10 лет к Европе летят целые две миссии(охуеть!) и у них там радары есть, которые могут через лёд пробиться, и еще там магнетометры будут, которые за счёт вариаций магнитного поля смогут определить глубину, солёность, хоть даже и состав подлёдного океана. Может при пролётах найдут естественные свежие кратеры со следами выбросов при помощи визуальных камер, может обнаружат криовулканизм какой через инфракрасные камеры.
Аноним 02/04/19 Втр 20:08:39 474748428
14822644698410.jpg (47Кб, 800x522)
800x522
>>474736
>10 лет
Чому так долго то? Он же еще лететь только будет лет 5. Уже старичком станешь к этому времени.
Аноним 02/04/19 Втр 20:19:25 474749429
>>474748
Радоваться нужно хоть какому-то результату. Кто-то ждал даже эти миссии очень долго. Знаешь как глаза занозит от вояджеровых 2 и галилеевых изображений, который до дыр досмотрены? Где даже мелкая точка в один пиксель может быть сложной географической структурой размером в несколько километров, а может быть просто убитым пикселем и шумом? Теперь хоть какая-то чоткая гарантия есть, что будут миссии, не одна, а две, а может даже три и посадят на Европу ландер, а может туда еще и Китай пустит зонд и тоже поснимает.

Для сравнения миссии к Урану или Нептуну может еще лет 30-40 ждать придется. Вот где уныние и безнадёга. Ну и всякие Эриды, Квавар, Седну, Хаумею наверное вообще в ближайшие 70 лет лишь из телескопов наблюдать точкой, вот до чего уж точно не доживем.
Аноним 02/04/19 Втр 20:28:08 474752430
world1598.jpg (1913Кб, 3264x2448)
3264x2448
>>474714
>Тогда звезда Полынь падёт на источники вод, и третья часть вод сделается полынью
Это куда должна Звезда-Полынь упасть, чтобы треть воды стала полынью? Очевидно же, что речь не обо всей воде, а о пресной, ибо морскую человек для еды-воды не использует.
Аноним 02/04/19 Втр 21:14:32 474758431
>>474749
блядь, нет бы сделали аппарат с солнечным парусом, да хуярили его мегаваттным лазером без остановки. Разогнали бы до 0.1с и долетели бы за пару месяцев до юпитера. нет нужно говно жрать с медленными ракетами
Аноним 02/04/19 Втр 21:24:09 474761432
>>474758
а мегаваттный лазер в космосе где взять? Вон китайцы построят свою космическую электростанцию, может и пристыкуют к ней лазер когда-нибудь через 30 лет. А может тогда с китайцами нормально будут отношения строить и станция будет китайская, а пристыкуют лазер американский и будет нормальное международное сотрудничество в космосе. Мы можем лишь мечтать о таком сейчас.
Аноним 02/04/19 Втр 21:27:35 474763433
>>474761
нахуй тебе в космосе? с земли хуярь, размечтался тут.
Аноним 02/04/19 Втр 21:32:09 474771434
>>474763
Земля вращается, аппарат в космосе двигается по орбите и на большой высоте, представляет из себя крохотную мишень, между вами атмосфера с парами воды и прочей газовой хуйней. Чтобы разогнать до 0.1с нужно долго в фокусе держать, а как ты это сделаешь, если Земля вращается, цель пропадает из фокуса и удаляется? В космосе всё проще, кроме охлаждения и постройки.
Аноним 02/04/19 Втр 21:44:34 474777435
15535176883690.jpg (146Кб, 600x870)
600x870
В п/sci/не нету Треда Тупых Вопросов, поэтому спрошу тут.
Ещё в школьные времена заинтересовала меня фраза, обранённая физичкой, мол, отдельный субатомные частицы, типа одиночного протона или электрона долго не живут. А что с ними, собственно, происходит? И типа, даже одиночные атомы тоже живут не долго, и речь даже не про какие-нибудь радиоактивные изотопы с большим числом нуклонов где-то в лантаноидах, а вообще, просто обычные атомы, хлор там, натрий, кремный. Куда они деваются? Рассасываются гипернаноатомарным излучением, аналогичым излучению Хокинга для чёрных дыр? А почему тогда атомы-социобляди, находящие в куче себе подобных живут себе и не рассасывают? Вот взял я кусок чистейшего золота, без единого инородного атома, на пару грам с каким-нибудь удобным числом атомов например сто тыщ мильёнов мильярдов, положил его на стол и этот кусок у меня лежит. Возьму его через 80 лет, позырить, а у него те же сто тыщ мильёнов мильярдов атомов, или он уже станет легче, потому что его атомы некоторым числом аннигилируются?
Аноним 02/04/19 Втр 21:45:03 474778436
>>474771
>Земля вращается
И что? Компьютерное наведение, ничего сложного.
>представляет из себя крохотную мишень
Можно немного рассеивать лазерный луч чтобы аппарат точно попал в пятно света.
> между вами атмосфера с парами воды и прочей газовой хуйней.
Ну возьми гигаваттный лазер, в чем проблема?
>нужно долго в фокусе держать
Поставить 3 лазера равномерно по периметру земли. Каждый отрабатывает по 8 часов в сутки.
>В космосе всё проще
Наоборот сложнее. Где ты возьмешь столько электричества? Даже пара десятков киловатт уже проблема. А на земле электричество почти дармовое и не надо тысячи тонн выводить на орбиту.
Аноним 02/04/19 Втр 21:54:38 474783437
>>474777
Это только нейтрона касается, он быстро распадается на протон и электрон бета-распадом. Протон стабилен, электрон тоже. Некоторые атомы сами по себе распадаются со временем, какие-то быстрее, какие-то дольше. Золото стабильно, по крайней мере обычный изотоп.
Аноним 02/04/19 Втр 21:58:39 474784438
>>474778
>Где ты возьмешь столько электричества? Даже пара десятков киловатт уже проблема.
Роскосмос вот-вот запустит ядерную установку мегаваттного класса на ракете "Енисей" с космодрома "Восточный".

А если серьёзно, то за ближайшие 10 лет мне кажется и правда запилят уже космические ядерные реакторы и будут летать на ионниках, да и лазеры тоже попробуют, как демонстрацию технологии в рамках проекта Breakthrough Starshot.
Аноним 02/04/19 Втр 22:02:10 474785439
>>474784
>Роскосмос вот-вот запустит ядерную установку мегаваттного класса на ракете "Енисей" с космодрома "Восточный".
Ага, запустят в очередном низкополигональном мультике, нарисованным каким-нибудь студентом, а выделенное бабло как обычно осядет у нужных людей в карманах.
Аноним 02/04/19 Втр 22:05:43 474787440
mmm.jpg (43Кб, 797x608)
797x608
>>474778
Тот случай, когда даже лень объяснять. Никто не будет с тобой играть в зебру из зелено-черного текста, если ты выдумываешь по 3 гигаватных лазера по периметру Земли, думая что это какой-то спор.
Аноним 02/04/19 Втр 22:24:03 474792441
>>474787
А в чем проблема?
Аноним 02/04/19 Втр 22:35:31 474797442
Глупый вопрос может, но всё же.
Орбита луны наклонена же относительно экватора Земли? Если да, то как определить её позицию в период времени-то (только относительно центра Земли) отстоящий от сейчас на вот столько-то часов?
Нашёл на https://space.stackexchange.com/questions/8911/determining-orbital-position-at-a-future-point-in-time
Относительно понятный ответ (на первый взгляд), где даётся ссылка на таблицы слаборатории реактивного движения со значениями + вводятся эпохи относительно 2000 года Юлианского календаря. Не совсем понимаю, как мне эти эпохи подскажут то, где будет Луна через, скажем, 10 часов.
Аноним 02/04/19 Втр 22:37:55 474801443
>>474792
Да ни в чём - почему бы вообще не построить тысячу петаваттных лазеров по экватору Земли, хули. Легко, без проблем, хоть завтра, в центр Галактики полетим, на коне, я и ты. Я капитаном будут. А потом салют в нашу честь.
Аноним 02/04/19 Втр 22:44:30 474805444
>>474777
С распадом простона не всё так однозначно. Это, как помню, чисто теоретическая возможность вроде. Вроде как может и распадаться, но вероятность низкая, условные 1 из 10e35 (на самом деле вроде степень от 30 до 40, помнил бы я точно) для одного протона в течение года. То есть ничтожно.
Кстати, сцена распада последнего протона во вселенной есть в futurama. А её правдивость (абсурдность) мы знаем. Эксперементы ведутся, но результат отрицательный пока.
Аноним 02/04/19 Втр 23:52:59 474823445
>>474797
С какой точностью надо?

Если вообще похуй на точность и можно считать орбиту Луны круговой, а наклонение постоянным, то по окружности Луна сдвинется на 360×t/T градусов (где t-время, а T-период обращения Луны), а ее отклонение от плоскости экватора будет изменяться по синусоидальному закону, где начальное положение равно A×sin(t0/T), а конечное A×sin(t1/T), где А-максимальная амплитуда, а t0 и t1 — время от момента прохождения восходящего узла.

Если надо точнее, то там настолько дохуя элементов в формуле будет, что лучше уж таблицы возьми. И эксцентритеты, и прецессии, и хуицессии, и узлы орбиты ползают, и орбита сама постоянно форму меняет, лучше просто не заморачиваться.
Аноним 03/04/19 Срд 00:00:16 474825446
>>474823
Похуй на точность. А почему t/T? Значение же всегда у нас <= 1 будет.
Аноним 03/04/19 Срд 00:06:00 474827447
>>474825
Потому что это то, на сколько градусов сдвинется Луна, если она несколько оборотов провернулась, то, понятное дело, и смещение будет больше 360 градусов. Если нужно только положение, то возьми остаток от деления на 360.
Аноним 03/04/19 Срд 00:28:19 474831448
>>474827
Вот к тому же глупому вопросу (воистину, правильный вопрос - половина ответа - знал бы до этого по sgp4, спросил бы у гугла).
А упрощённая модель sgp4 подойдёт для определения того, что я спросил? Или она только для маленьких объектов годится?
Аноним 03/04/19 Срд 08:22:51 474868449
>>474801
Ты так говоришь, будто мощный лазер это какая-то проблема в 2019 году. Ну поставь рядом 100 слабых лазеров. Будет тебе как один мощный.
Аноним 03/04/19 Срд 08:37:03 474871450
>>474868
Да, да, конечно. Можем вообще вместе с тобой организоваться. У меня есть мощный лазер для хозяйственных нужд, у тебя скорее всего тоже. Нужно еще 98 граждан найти, не сложно по сути, ведь по статистике на 1000 человек приходится как минимум 853 хозяйственных лазера мощностью от 100 киловатт. Вчера вот с пацанами направили мой лазер на Луну и как и предсказано, сигнал пришел назад от одного из рефлекторов, был пойман оптическим телескопом Серёги, который он купил на стипендию. Он фазирование вычислил по простой школьной формуле, и мы сразу поняли, что отразился от ретрорефлектора с Лунохода.
Я бы тоже телескоп купил, но вот недавно прикупил с тринадцатой зарплаты атомный микроскоп чисто по фану чтобы посмотреть на атомы после работы.
Аноним 03/04/19 Срд 08:48:23 474874451
f964e39a5dff81b8.jpg (27Кб, 598x430)
598x430
Аноним 03/04/19 Срд 09:53:52 474881452
15501284887841.png (397Кб, 465x604)
465x604
Что будет с пульсаром который вращается 599 раз в секунду, если он в миг остановится.
Аноним 03/04/19 Срд 09:53:52 474882453
>>474752
Куда-то в Южной америке. Амазонка (20% мировых запасов) с притоками (у некоторых по 1-2%) имеет отдалённые шансы потянуть на треть запаса пресных вод.
Альтернативы - Байкал и Великие озёра (у обоих тоже примерно по 20% мировых запасов, но нет таких мощных притоков).
Аноним 03/04/19 Срд 10:01:28 474883454
Аноним 03/04/19 Срд 10:07:33 474885455
>>474883
И его не расхуярит от инерции? А близ летающие объекты?
Аноним 03/04/19 Срд 10:52:30 474899456
Что известно про нортропграммановскую Зуму (кроме того, что написано в вики)?
Аноним 03/04/19 Срд 11:20:51 474908457
>>474899
Миссия секретна, так что неизвестно ничего.
Аноним 03/04/19 Срд 16:18:14 474957458
Что такое этот Гелий-3 и почему ради него хотят пылесосить луну?
Аноним 03/04/19 Срд 16:46:52 474960459
Поясните за ДНК, ведь это язык, почему бы не изобрести язык общения на основе ДНК? И не программировать своих животных?
Аноним 03/04/19 Срд 17:01:00 474962460
>>474957
Гелий третий это такой царь такой. Был гелий первый, хороший был царь, был гелий второй лучше первого, а вот третий гелий настолько охуенный был, что бог Зевс его на Луну поместил и его тело стало священным песком на Луне, теперь оно даёт силу могучую тем, кто его сможет правильно приготовить. Правда рецепт приготовления гелия третьего слишком сложный и никто не может его освоить.
Аноним 03/04/19 Срд 17:01:51 474963461
Аноним 03/04/19 Срд 17:17:14 474968462
>>474758
>Разогнали бы до 0.1с и долетели бы за пару месяцев до юпитера
Ну допустим разогнали. Как тормозить будем? Об Юпитер?
Аноним 03/04/19 Срд 17:18:28 474969463
>>474968
Спиралью вокруг планеты.
Аноним 03/04/19 Срд 17:46:33 474972464
>>474969
На скорости 0.1с выходить на орбиту? Лол, сайфайно
Аноним 03/04/19 Срд 18:17:21 474978465
001.jpg (41Кб, 1096x398)
1096x398
>>474968
>Ну допустим разогнали. Как тормозить будем? Об Юпитер?
Пикрелейтед. Везём с собой ещё одно зеркало, перед торможением отстыковываем. Лазор светит на зеркало, отражается и тормозит корабль. (Можно даже несколько переотражений сделать, усиливая тормозящий импульс, пока в красное смещение не упрёмся.)
Зеркало в процессе улетает нахуй на околосветовой, да и в рот его ебать. А корабль оттормаживается возле Юпитера.

>>474761
>а мегаваттный лазер в космосе где взять?
Скиф-ДМ, макет которого на Энергию вешали, на тогдашних технологиях 100кВт был. В принципе, мегаваттный очень недалеко от того, а современных солнечных батарей на мегаватт нужно всего 70кв.м. Я, правда, сильно не уверен, что мегаваттного лазера на 0.1с до Юпитера нам хватит, но один хуй проблему с торможением я только что решил, и открыл человечеству дорогу к звёздам.
Аноним 03/04/19 Срд 19:33:33 474981466
Нука, а вот допустим жахнули ядреными бомбами и наступила ядерная зима. А почему это будет зима - с понижением температуры, а не пекло как на Венере из-за парникового эффекта?
Аноним 03/04/19 Срд 19:42:18 474982467
Аноним 03/04/19 Срд 19:55:27 474983468
>>474978
>на мегаватт нужно всего 70кв.м
Толстишь? 1 кв.м дает около 200 вт мощности в лучшем случае.
Аноним 03/04/19 Срд 19:56:48 474984469
Как считать красное-синее смещение? Если например едешь на скорости 70% световой относительно источника света, то на сколько сместится длина волны этого света?
Аноним 03/04/19 Срд 20:33:50 474985470
>>474983
Ну да, объебался немного с размерностями. Вроде как на орбите 1367 Вт/м², а кпд хороших солнечных батарей сейчас 25% (есть до 44%, но ебически дорогие). http://energosberejenie.org/stati/samye-effektivnye-solnechnye-batarei
Тогда 2900кв.м. или квадрат со стороной 54м. Что в общем-то тоже хуйня.
Аноним 03/04/19 Срд 22:50:00 475001471
>>474981
Так-то есть теория, что парниковый эффект перевесит, и никакой ядерной зимы не будет. И даже весьма вероятно
Аноним 03/04/19 Срд 22:50:31 475002472
Правда ли что мелкий космический мусор может серьёзно повредить МКС?
Аноним 03/04/19 Срд 22:53:46 475003473
Аноним 03/04/19 Срд 22:59:51 475004474
>>475001
Ссылочку на раша тудей теорию в студию. Там же всякая пыль и сажа должна быть, которая как раз-таки солнечный свет и блокирует.
Аноним 03/04/19 Срд 23:05:36 475005475
>>475004
А эта пыль и сажа сама не нагревается, блокируя?
Аноним 03/04/19 Срд 23:16:56 475008476
>>475002
Может, впрочем по счастью для МКС ее трасса проходит в относительной безопасности но опасные моменты бывают
Аноним 04/04/19 Чтв 00:15:07 475018477
HypervelocityIm[...].png (647Кб, 795x841)
795x841
Аноним 04/04/19 Чтв 02:16:21 475030478
>>475005
Нагревается, но излучает-то не только дальше в Землю, но и обратно в космос. То излучение от Солнца, что не дошло до поверхности Земли - как раз и есть потери от "ядерной зимы".
Аноним 04/04/19 Чтв 02:44:19 475031479
>>475018
Пиздеж и постанова, от шарика бы ничего не осталось.
Аноним 04/04/19 Чтв 08:10:58 475067480
>>475031
Шарик для наглядности.
Аноним 04/04/19 Чтв 08:31:31 475069481
1554355880329.jpg (102Кб, 640x400)
640x400
У нас гипотетически есть мегаплотный белый карлик. У него же есть гравитационная линза? А если он станет черным карликом у него эта линза останется?
Это значит, что гипотетически могут существовать чёрные карлики с гравитационными линзами?
Аноним 04/04/19 Чтв 09:40:52 475076482
>>475069
Черных карликов не существует и не может существовать даже гипотетически, в нашу Вселенную их через триллионы лет только завезут. Нет таких механизмов, по которым они могли бы образоваться и остыть за считанные миллиарды лет, все белые карлики, возникшие за время жизни Вселенной, всё еще дохуя горячие.
Аноним 04/04/19 Чтв 10:52:50 475080483
image.jpeg (790Кб, 1004x1400)
1004x1400
>>475076
> Черных карликов не существует и не может существовать даже гипотетически, в нашу Вселенную их через триллионы лет только завезут
Аноним 04/04/19 Чтв 10:59:41 475081484
>>475076
Ну и подождём, не беда. Мне главное, что гипотетически возможно существование черной йобы с линзой, которая выглядит как чд, но ею не является.
Аноним 04/04/19 Чтв 11:16:53 475082485
>>475081
Что еще за черные карланы? Разве такие массивные звезды, которые могут в линзы, не въебывают, когда выгорают?
Аноним 04/04/19 Чтв 12:08:31 475085486
>>475069
В любом случае, эффект линзирования там будет сильно меньше, чем у черной дыры из унтерстоляра.
Аноним 04/04/19 Чтв 12:13:38 475087487
>>474968
Развернуть корабль, хотя бы даже вонючкой, и подлороги пердеть против хода.
Аноним 04/04/19 Чтв 12:14:46 475088488
>>474978
>Pic
Ты себя за волосы
Аноним 04/04/19 Чтв 12:24:43 475090489
>>475069
Гравитационно линзирует все, даже планеты. Вопрос только в "фокусном" (хотя там нет настоящего фокуса) расстоянии. Ну и еще в собственных размерах и светимости объекта. Чем меньше размеры - тем ближе свет может пройти и больше отклониться при той же массе. Собственная светимость мешает наблюдать линзированный свет. Хотя, например, у Солнца гравитационное линзирование все равно наблюдают.
Аноним 04/04/19 Чтв 13:31:25 475101490
>>475090
Ты точно не путаешь насчёт наблюдений линзирования Солнца? Там вроде 550 ае нужно, не?
Аноним 04/04/19 Чтв 14:22:38 475113491
>>475101
Ну, там не совсем "линзирование" получается, просто гравитационное отклонение света. Но природа явления одна и та же, тащемта. Просто на 550 а. е. (видимо) свет, отклоненный с разных сторон Солнца сойдется в одну точку.
Аноним 05/04/19 Птн 04:53:39 475238492
>>474467
Объясните гифку.
Аноним 05/04/19 Птн 05:52:41 475240493
001.jpg (110Кб, 1078x916)
1078x916
>>475088
>Ты себя за волосы
Нет. У нас тут обычный третий закон ньютона, когда два тела расталкиваются внешним источником энергии. Суммарный импульс системы не изменяется. Свободно летящее зеркало выступает в качестве рабочего тела. По сути это реактивное торможение, только с возможностью разместить источник энергии позади объекта.

Здесь есть лишь две изюминки:
1. Расталкивающая зеркала сила может быть многократно увеличена переотражениями. Если бы у нас были идеальные зеркала, пакет фотонов мог бы создать на них сколь угодно большую силу в статике. В динамике это превращается в понижающую передачу, ограничиваясь законом сохранения энергии.
2. На достаточно большой скорости масса зеркала может быть много меньше массы корабля, и при этом корабль будет тормозить быстрее, чем разгоняется зеркало. Связано это, очевидно, с так называемым релятивистским увеличением массы (на самом деле не массы, а импульса).

>>475238
Соотношение размеров стран на картографической проекции и в реальности. На самом деле Россиюшка не циклопическая довлеющая над всеми материками хуёвина, а куда более скромная ненамного больше Китая. А её климатически пригодная для обитания область так вообще размером со сраный хохлостан. Как страшно жить.
Аноним 05/04/19 Птн 10:18:19 475263494
Поясните за воду и давление. Почему если я погружаюсь в озеро на метр, мне уже на голову давит, подводные лодки на сотни метров погружаются со скрипом, а рыбы на дне океана норм плавают?
Аноним 05/04/19 Птн 10:20:16 475265495
>>475263
Может в рыбах давления дохуя внутри, чтобы их не плющило?
Аноним 05/04/19 Птн 10:21:12 475266496
>>475265
А чем их так распирает? И почему они вне воды тогда не взрываются?
Аноним 05/04/19 Птн 10:30:14 475269497
>>475266
Вода и распирает, вестимо. Но если вытащить на поверхность какого-нибудь ебанутого удильщика с 5км, то он должен будет лопнуть.
Аноним 05/04/19 Птн 10:35:33 475270498
>>475269
Надо еще быстро вытаскивать, а если медленно вытаскивать, то он будет вдыхать, пить и выплевывать воду, адаптируется плавно и просто разбухнет разрыхлится
Аноним 05/04/19 Птн 12:16:52 475285499
>>475269
Напомни, на как изменяется объём воды при изменении давления. Плиз.
Аноним 05/04/19 Птн 12:17:55 475286500
>>475285
Кстати, я слышал что вода якобы не сжимается. Поясните?
А что будет если куб воды налить в суперкомпрессор и сверху кинуть миллиард тонн?
Аноним 05/04/19 Птн 12:39:27 475288501
>>475286
Вода сжимается, всё сжимается под давлением.
Аноним 05/04/19 Птн 12:48:50 475289502
>>475285
Никак. Вода это не газ.
Аноним 05/04/19 Птн 12:55:31 475290503
>>475286
Коэфициент очень низкий, а так сжимается.
На каждую атмосферу повышения давления вода сжимается на 4.6e–10 объёма.
Нормальное давление - где-то 10 тонн на метр.
Соответственно, в кубическом чане метр на метр на метр мы добавим 100 миллионов атмосфер, из-за чего вода сожмётся на 4.6 е-4 объёма, или на 0.046%.
Аноним 05/04/19 Птн 13:45:24 475294504
>>475289
>Никак
Как. Вода сжимается, хотя и очень слабо. Где-то пробегал факт, что если бы вода была абсолютно несжимаемой, то уровень океанов поднялся бы на 30 метров, что ли.
Аноним 05/04/19 Птн 13:54:46 475297505
>>475294
В вопросе про рыб можно принебречь.
Аноним 05/04/19 Птн 14:01:13 475301506
>>475297
>принебречь
Уроки-то сделал?
Аноним 05/04/19 Птн 14:10:29 475302507
>>475301
>мам смари я до грамматики на двачах доебался
Аноним 05/04/19 Птн 14:12:05 475303508
>>475290
>в кубическом чане метр на метр на метр мы добавим 100 миллионов атмосфер, из-за чего вода сожмётся на 4.6 е-4 объёма, или на 0.046%
Нет, она превратится в лёд-XI или лёд-X
Аноним 05/04/19 Птн 14:59:13 475305509
>>475294
Под давлением в 25 000 ат метровый водяной столб сжимается до высоты 65 см (опыты Джеймса Бассета, 1933 г.).
Аноним 05/04/19 Птн 15:08:13 475308510

>>475305

и где они взяли 25000
Аноним 05/04/19 Птн 15:15:17 475310511
>>475308
Искусственно умеют создавать и гораздо большие давления, см. https://ru.wikipedia.org/wiki/%D0%AF%D1%87%D0%B5%D0%B9%D0%BA%D0%B0_%D1%81_%D0%B0%D0%BB%D0%BC%D0%B0%D0%B7%D0%BD%D1%8B%D0%BC%D0%B8_%D0%BD%D0%B0%D0%BA%D0%BE%D0%B2%D0%B0%D0%BB%D1%8C%D0%BD%D1%8F%D0%BC%D0%B8
>таким образом могут быть достигнуты давления до нескольких миллионов атмосфер
Хотя, конечно, это для крошечных объемов веществ, но это на порядки больше 25 тыщ.
Аноним 05/04/19 Птн 15:27:58 475311512
>>475310
чтобы не засирать пост такими длинными ссылками можно, перед русским названием поставить пробел, скопировать сюда и затем удалить его
Аноним 05/04/19 Птн 16:12:56 475318513
Аноним 05/04/19 Птн 19:53:00 475339514
1475453390938.jpg (127Кб, 1160x629)
1160x629
Планеты в солнечной системе образовались из протопланетного облака пыли, которое окружало молодое Солнце, так? А может быть так, что звезда вроде нашего Солнце будет проходить через облако пыли в космосе и приобретет массу материала для образования новых планет? Или же солнечная радиация просто создает вокруг него щит, который всю пыль нахуй сносит и для образования планет может использоваться только начальный материал, который присутствовал лишь до зажигания термояда в звезде?
Аноним 06/04/19 Суб 01:33:56 475368515
Лазор, светящий в космосе, будет иметь реактивную тягу?
Аноним 06/04/19 Суб 02:28:17 475373516
>>475368
Конечно, будет. У излученных фотонов импульс есть, значит, и у лазера будет такой же, только в противоположную сторону.
Аноним 06/04/19 Суб 08:28:54 475380517
Что именно может пролить свет на загадку темной материи? Чего ученым не хватает? Вычислительных мощностей, энергии, каких-то специфических данных или техники?
Аноним 06/04/19 Суб 10:20:28 475388518
>>475380
>Чего ученым не хватает?
Грантов им не хватает. На самом деле любому адекватному человеку очевидно, что темная энергия и материя это математические костыли, которые с разных сторон подпирают противоречащую наблюдениям Эйнштейнову физику. Причем "вес" этих костылей в десятки раз больше наблюдаемой ИРЛ Вселенной. Отака хуйня, малята. Зачем это нужно? Потому что лучше этой физики у нас все равно ничего нет.
Аноним 06/04/19 Суб 10:27:08 475390519
>>475263
Потому что ты живешь на поверхности, и внутри тебя есть полости, в которых давление ~атмосферное. Рыбы рождаются и живут всю жизнь под давлением в сотни (тысячи) очков, и соответственно внутри них давление такое же, поэтому им и норм. Если тебя в батискафе довезти в барокамеру на морском дне, где плавно поднять давление до уровня морского дна, то наверное через какое-то время ты сможешь там плавать с легким аквалангом и не испытывать никакого дискомфорта.
Аноним 06/04/19 Суб 15:00:33 475403520
>>475390
>Если тебя в батискафе довезти в барокамеру на морском дне, где плавно поднять давление до уровня морского дна, то наверное через какое-то время ты сможешь там плавать с легким аквалангом
Не сможешь. У глубоководных организмов метаболизм другой: другие белки, ферменты, вот это все. Как минимум, у тебя нарушится половина метаболитических реакций и организм самоотравится за пару часов. Но предположу, что дело будет даже хуже, и какие-то белки при таких давлениях просто развалятся, возможно будут идти какие-то вообще непредусмотренные химические реакции.

Еще, пожалуй, газ из штатных полостей под безумным давлением будет быстро диффундировать куда не надо. Кислород, тащемта, тот еще яд (в норме он практически всюду в организме химически связан), и должен человека очень быстро похерить.
Аноним 06/04/19 Суб 15:38:56 475407521
Перекатывайте, аутисты
Аноним 06/04/19 Суб 16:30:26 475416522
>>475407
Возьми и перекати.
Аноним 06/04/19 Суб 16:47:37 475417523
Поясните вот что, если солнце светит жёлтым светом, почему я вижу всё разноцветным, а не градациями жёлтого?
Аноним 06/04/19 Суб 16:57:49 475420524
>>475417
Ну ващет солнце белое, а если присмотреться к спектру, то слегка зеленоватое, желтым его делает атмосфера.
Видишь ты разноцветное потому, что предметы на которые ты смотришь, поглощают все цвета, кроме того, в которые они "окрашены", этот цвет отражается и попадает тебе в глаза.
Аноним 06/04/19 Суб 16:58:03 475421525
images.jpg (5Кб, 275x183)
275x183
Аноним 06/04/19 Суб 17:03:54 475424526
>>475417
>жёлтым
На самом деле зеленым.
Аноним 06/04/19 Суб 20:59:25 475461527
Спс
Аноним 09/04/19 Втр 16:33:20 476025528
Настройки X
Ответить в тред X
15000 [S]
Макс объем: 40Mб, макс кол-во файлов: 4
Кликни/брось файл/ctrl-v
Стикеры X
Избранное / Топ тредов